You are on page 1of 244
Page | of 4 Before the exam . First of all, keep in mind and try to stay calm about the fact that you will be just a number in @ herd being put through the thing, you completely give up your individuality when you take the exam, Try not to take that personal. Tt is just the way it is. Everything is well organized and timed; the CSA proctors are very friendly but also efficient. Try to arrive at the test center 30 minutes early, even though they will only put you in a room together with everybody else and wait until everyone has arrived. They start the registration not before 8.30. You will just sit around getting even more nervous if you get there early. At 8.30 they start registering, you have to present your permit (that you got after you confirmed your exam date via phone or internet) and one ID, like passport or driver's license, if you do not have the permit with you, no sweat, you will just need two forms of ID. Your name is on their list. Then you get an ID badge, (which is the pink littie slip you sent in with a photo when you applied for the CSA, I was very embarrassed because my picture on it was terrible), to put on your chest pocket of the lab coat, and also a big arm band with a number on it to wear on the left upper arm-something | found very disturbing Then they hand out a confidentiality agreement, two copies, one for you to keep after they hand it out after the Exam, and 11 pages of note paper for the encounters and a pen that you are only to use during the whole time. Tam not kidding, The room was packed with 22 people, a group of 11 in each blue and pink armband division, It took a while until { realized that they would run two groups parallel, meaning two sets of 11 5.P's for each group. Around 9.00am everybody was registered and got the clipboard with pen and notepaper. Everybody was asked if we had our equipment, they even offered lab coats, stethoscopes and watches in case someone forgot about it. We then entered the orientation room, had to sit down in order of the numbers at a large U form of tabies. We were asked to take everything out and put on the tables what me might need during the exam as everything else would be stored and tocked away curing the exam, the storage room is small, no place for luggage, just tiny little cubicles, numbered again, just for backpacks. Page 2 of 4 + Aslide show orientation started, with familiar and some new info: 1. You are allowed to palpate axillary and inguinal lymph nodes including the femoral pulse. 2. Postural signs and BP repeat can be ordered in the work-up. Don't waste your time for measuring those things. «Although it was emphasized that using the gloves is fine 1 would still recommend washing the hands, they have only one size of gloves, and seriously, in my whole career as a doctor T have never seen anybody using gloves to palpate the abdomen ar percuss the lung... 80 why do it now, I was able to wash my hands within 20 seconds. + At one side of the orientation room an exam room was set up and there was ample opportunity to use the instruments, try out the bench, the drape, the forks etc. Questions were welcome and answered nicely. «Then after a last bathroom break we finally started, it was about 9.50am already = The only thing you can bring yourself is the lab coat and stethoscope, everything else is provided, © Here ig what is presented in the room, how anybody shall have time to use all of this is beyond my imagination though: + Snellen's visual chart at the wall; two tuning forks with different frequencies; cotton swabs and toothpicks; tongue depressors; reflex hammer; ophthalmoscope; otoscope; blood pressure cuff The actual exam + There is a long but narrow corridor of about ten feet width, to each long side about 15 doors. «The examinees face the doors. Between the doors there are small workstations settled, where you will write the patient note before walking up to the next door. + When you face the door, there is a plastic box on the door. It does contain the doorway info. When everybody Is settled and ready, you get the signal to start your encounter, «This is where the fifteen minutes start!!! You have to open the box at the door and read the Page 3 of 4 doorway Info. + The doorway info is scarce, it will tell you the full name, age, setting (ER, office}, chief complaint and vitals (BP, pulse, temp, respiration) «At the bottom they tell you what to do, just like In the info baoklet, The time you spend reading and taking notes DOES count te the 15 minutes. + Do not rush into the room, because once you enter, you will be totally occupied by the SP. + Try to spend about 45 seconds in front of the door to make @ mental nole uf differential diagnosis. This is extremely important. ‘= When you are in the room time flies. So budget your time accordingly. + You do NOT have to write everything down, there is a second copy of the doorway info in the room, + All patients will be sitting on the exam table in their gown when you enter the room. ‘+ In the tiny rooms there is @ chair right in front of the bench where you can sit down. + Also, to the side of the bench there is a little stool with the drape folded on top of it. In the corner there is @ little workstation with computer on it, but net for you, it Is for the $.P's to grade your performance. + At ten minutes you hear the signal of 5 minutes remaining for this encounter. Then at 15 minutes there is a signal that this encounter is over and you heve to leave the room. Still, you do not have to just cut off the sentence and run out, you still can finish the encounter with a quick bye-bye, after about ten seconds they nicely knock on the door te get you out. + Right after the 15 minutes the ten minutes for the patient note start. After 8 minutes, and this was very helpful, there is a signal to let you know that there are 2 minutes remaining for your note. + After the ten minutes you have to put down the pen and wait for the proctors to collect the patient nate and the piece of notepaper. While writing the note you can still get up, open the doorway info again and read it, I did that twice and it was fine to do so as tong as you slide it shut again, Not Page 401 4 Then the next encounter does start only after everybody has walked up to the next-door and positioned again. This gets into a nice routine and it is over sooner than you think. After 4 patients there was a 30-minute lunch break, then after another 4 cases @ 15-minute break ‘The food there was excellent, very good cater service, even vegetarian. They offered sandwiches, fruit, cookies, salad and bread. A feast if your stomach weren't so occupied with nervousness. Also, there is no need to bring drinks; they served cold and hot water, tea, coffee, soft drinks. Seriously, you do not have to worry about getting food to bring. It was more than sufficient and everybody thought so. After the exam they ask you to complete an anonymous questionnaire, but still they ask you to put your USMLE number on there. We have no clue about this, why give them my number if it is supposed to be anonymous? They ask general things about your opinion of the exam, and also about your ideas of doing the notes at the computer. ‘The exam usually finished at 3.45pm for morning session. . Most of the peopie have no idea how they did, there is just no time at all to nearly get everything asked and done the way you would like to...just remember this...no matter what you did, you will feel frustrated afterwards. It ts completely normal, 1 seriously think this is one of the things they test, how well you can cope with time pressure and frustration... ‘The proctors were very warm and effective; they really tried to make the thing as pleasant as possible. : This Is the actual exam picture when we took the exam. We greatly appreciate # you can provide changed information after your exam through our feed back form. ©2002-2003 Alll Rights Reserved www.usmteworld.com History Taking Page 1 of 16 History Taking + The most difficult part of the exam is time maintenance, You have to do everything in 15 minutes. So to make everything in 15 minutes we made a very good proforma to pass in this exam, Introduction and greeting: + Hello Mr. xyz, good morning or good afternoon, I am Dr. xyz, it is nice to meet you (shake hand). «Next ask - "What brings you to see me today” or "What brings you to come in today?". * Then the SP will tell you the chief complaint (C.C} ‘+ Then ask one open ended question - "Could you please describe me what exactly is going on ‘er more about your problem.” Then he will tell you only minimal but some extra history, but remember he will never tell you the important facts that you have to ask. # Then you have to proceed like this Location: « First thing that you have to keep in mind is: "Where is the problem?" ("Can you please show me where exactly the pain/problem is?"). If the C.C is a pain you can ask "Can you show me the exact spot with your finger?". Onset & duration: Always ask for the onset and duration of the problem. So that you can know whether the problem is acute or sub acute or chronic. * Duration: Ex. "When did it first start or when did you first notice?" Depending upon the complaint. «If the C.C is chest pain ~ "When did it first start?” + Ifthe C.C is vaginal discharge “When did you first notice?” History Taking Page 2 of 16 * Then ask about the onset “Was all of sudden in onset or gradual in onset? « Then you have to ask follow up questions regarding the C.C - Intensity: + Always one should ask about the intensity or severity of the problem especially if the complaint is pain. * Ex. "Ina scale of 1 to10, if 1 is the least painful and 10 is the most painful, which number would describe your pain?" Or ‘How would you grade your pain on a scale of 1 te 10?" + Remember they already know some medical knowledge, they will immediately tell you the number, Just ask like that . « If the chief complaint is not a pain you can assess the severity by asking questions like these. “How bad is it?”, "Does it interfere with your daily activities?", "Does it disturb your sleep?". Quality: © Quality of the pain may tell you the problem like burning type of pain in acid peptic disease and GERD. « Ex. "How would you describe your pain?" or "What does it feel like?" "I mean is it crampy? sharp? thrabbing? burning?” Frequency: « Always ask about how frequent the problem is? + Ex." Is it constant?" or "Does it come and go?" If it is intermittent, "How often does it cccur?", "How long does it jast?", "How do you feel between attacks?” # This is usually asked if the complaint is pain. «Ex. "Does it move anywhere?" or "Has it changed location?" itating factors: History Taking Page 3 of 16 «You may get a clue from aggravating and precipitating factors like if having food aggravates the epigastric pain i.e. most likely a gastric ulcer. «xl: "What were you doing when it first began?", "Have you ever found anything that makes your problem/pain worse or bad? « Ex2: "Do you have any idea of what might have brought this on?" or “What brings it on?" Relieving factors (alleviating) factors: + Along with aggravating and precipitating factors this will also help you in making diagnosis in certain conditions like having food will relieve pain in duodenal ulcer, «Ex, "Have you ever found anything that makes your pain or problem better?", Continue “Have you treated yourself?", "Has the treatment helped?”. Associated problems: + Here again you can ask one more open ended question. ‘+ Ex, Do you have any other problems that you would like to tell? (or) Do you have any other symptoms beside "Chest Pain” etc.? + When you ask the SP like this, the SP may ask you "Like what?" That's why you have to continue with all the pertinent positives and negative symptoms. Fever: » Ifyou think the symptoms are due to infectious origin, or the suspecting condition is associated with fever you have to ask about the fever. + "Do you have fever?" or "Have you had fevers?" @ If yes then "How long have you had fever?" "Is ita continuous or intermittent fever?” © "Is/Was it associated with chills, is/was there any sweating?” + "Did you measure the temperature’ "Was it a low grade fever or high grade fever?" History Taking Page 4 of 16 Cough: « "Do you have a cough?” # "Is it a dry cough or productive cough?” or "Do you bring up sputum?" « [fit is productive then "what color is/was It?" ‘« "Is/Was there any blood in it?” « “Is/Was it foul smelling?” '¢ "How much is/was it?” “Is it a tea spoon or table spoon or a cupful?" ( for quantity assessment always use tea spoon, table spoon and a cupful. Even for bleeding per rectum ). * Inall chronic cough patients don't forget to ask about HIV status and tuberculosis. They will not tell you until you ask about his HIV status*, You should also have to ask about drug intake especially about the use of ACE inhibitors*. Shortness of breath: + "Have you ever had any problems with your breathing’ lave you been wheezing?” (They know what is wheezing). * "How far do you walk on level ground without having breathing problem?", "Do you have to stop to rest to catch your breath?” + “Have you had any attacks of breathlessness in the night?” (PND) ‘+ "Do you need to sit-up to have asleep?" (orthopnea) or “Do you have trouble lying flat in bed?" Nausea and vomiting: * "Have you felt nauseated?/Do you feel nauseated?" » "Have you had vomitings or did you throw up?" « If yes then "How many times? What does the vomitus look like? What color was it? Was there any blood?” History Taking Page 5 of 16 ‘+ Ask the nature of vomiting like projectile etc. Headache: ‘+ "Do you have any headache?" Edema: ‘+ "Do you have any problems of swelling of arm/legs?” or "Do your ankles swell?” « If yes ask ” where did you notice it first? was it on the face or leq?” + Ask them about any diurnal variation ( "is the swelling mere during morning or evening?") Thyroid: » "Have you ever had problems adjusting to temperatures?” + “Is there any change in your voice recently?” (hoarseness in hypothyroidism) + "Have you noticed any change in your bowel habits recently?" (constipation in hypo and diarrhea in hyperthyroidism) « "Has your weight changed anyway lately’ Past Medical History: + Here we give you an example of how to elicit past medical history (This would mainly give you an idea of how to frame questions and save time). «You have to use transition sentences often during this part of history taking. Below is an example of a transition question (you would tell the patient about what you are going to ask instead of directly jumping into other topic} ‘© Example: “Ok Mr. Brown now I would like to ask few questions regarding your past medical health, fs that ok with you?” Previous episodes of chief complaint: Ex : "Have you ever had similar problems in the past?” History Taking Page 6 of 16 Allergic history: * Bear in mind that most of the SPs have some sort of allergic history though it is not related to the chief complaint. Therefore you have to take the ailergic history. (In brief if the patient's complaints are not mainly related to alleray like case of menopause, psychiatric case....) Just ask “Are you allergic to anything?" or "Oo you have any allergies?" « If the case is retated to allergy, ( C/O Shortness of breath, rash, arthritis..... ) then you can elicit the history in the following way: “Are you allergic to pets? (Give a pause and ask for the next allergen) Drugs? Any specific foods? Cold and dust allergy?” « If the SP gives you any positive history then ask follow up questions: Start off with an open- ended question like: “Could you please describe more about your allergic problem?" * If he doesn’t open up properly then ask the following questions * How often do you get allergic episodes? Are you taking any medication for that? What kind of allergic reactions did you have?" Medical problems in the past: ‘+ In general for any case you will have to ask history of high blood pressure (do not use medical terminology \ike hypertension) and diabetes (this is’ accepted). * In cases related to specific systems the following questions are to be asked: * CNS - "Have you ever had any stroke?" "Do you have any history of migraine headaches?" "Have you ever had any seizures? * CVS- "Have you ever had heart problems like heart attack or heart failure?” * RS - "Have you ever had tuberculosis?", "Do you have a history of asthma?", “Have you ever had any lung probiems?” * GIT- "Have you ever had stomach problems or ulcers?", "Have your ever had any problems with your galibladder or liver?" © RENAL. had any kidney stones?", “Have you ever had any problems with your prostate?” lave you ever had any history of kidney infections?” "Have you ever + THYROID PROBLEMS (Never forget to ask about thyroid as many cases (Ex: SP History Taking Page 7 of 16 with C/O weight loss/weight gain, Depression, Amenorrhea etc} are related to thyroid. They will be reacy to tell you if you just ask them. They might also give you directly the name of the disease. like Hashimoto's Thyroiditis, Goiter etc. » Cancers - "Have you ever diagnosed with any type of cancer?" Hospitalization: ‘+ Ask about any past h/o hospitalization, trauma and h/o surgeries. ‘+ “Have you ever been hospitalized?", What for?" "When?". » “Have you ever had any surgery?", "what for?” "When?". + "Have you ever been invotved in a serious accident?", "Did you break any bones?", "Did you have any serious head injury?” Urinary complaints: * If the case is not related to urinary system just ask : "Have you had any problems with your urination >” or "Do you have any trouble with your urine?" « If related to Genitourinary system then take a detailed history. # H/O Burning micturition ("Have you had any burning sensation after you pass, urine?") + H/O Urgency (‘Do you have to rush te the bathroom to pass urine?") *H/O Frequency/Nocturia (“How frequent do you have to pass urine? Do you have to wake up in the night to pass urine?") + H/O Hesitancy ("Do you have to wait before you start urination?") ¢ H/O Hematuria ("Did you notice any blood in your urine?"} ‘¢H/O Pyuria ("Was there any pus in your urine?"} # H/0 Straining ("Do you have to strain during urination?") « H/0 Changes in stream of urine ("How is your flow of urine? Is #t continuous or is History Taking Page 8 of 16 there any dribbling after urination?") + H/0 Incomplete emptying ("Do you feel fullness of bladder even after passing urine?") ‘+ H/O Incontinence ("Have you ever passed urine with out your knowledge?") Gastro intestinal problems: If the case is not related to GIT then just ask: " Have you ever had any problems with your bowel movements?” ‘* "How often do you move your bowels?” » "Have your bowel movements changed?" + "Are they hard or soft?" "What consistency?” “What color?" « "Have you noticed any black or tarry stools?" Sleep: » Enquire whether he has any problem with his sleep ("Do you have any problem with your sleep?”) » Ifso ask whether he has difficulty falling asleep? Cr maintaining sleep? Or early wakeup? « This is mainly required in all psychiatric cases, Family History: + Before taking the history fet the patient know that you would be asking him about his family health, .e., pose a transition question. © "Ck Mr, Brown now I would like to ask few questions regarding your family's health, Is that ok with you?" And continue as follows * “Does any one in your family have similar problems?” * “Are your parents living?” History Taking Page 9 of 16 «If SP says YES, then ask "How is their health?" «If SP says NO, show some empathy like "Oh! I am sorry to hear that, Could you please tell me the cause of their death?” «If necessary ask for the family history of diabetes, high blood pressure, stroke, heart problems. ‘Obstetric and Gyn History: + Before taking the history let the patient know that you would be asking about her Obstetric and Gynecological history.( So you will be posing a transition question here.) © Ok Mrs, Smith now T would like to ask few questions regarding your gynecological health, 1s that ok with you?", continue as follows « If itis not a Obstetrical/Gynecological case just ask : 1. “When was your last menstrual period?”, 2. “Are/Were your cycles regular?”. If it is a OB/Gyn case enquire about * Menarche "How old were you when you had your first period?” * "Are your periods regular?” © "How many days does your period tast?” “Have you ever bleed between cycles?" * "How many pads do you use ina heavy day?" "Do you have abdominal cramps/pain with your periods?" ‘+ "Did you ever notice any bleeding after intercourse?" ‘© "When was your last menstrual period?” Vaginal discharge: History Taking Page 10 of 16 » "Have you ever had any vaginal discharge?" + If YES, then ask “What is the color of the discharge? Does it have any bad’ odor? Do you have any vaginal Itching?" * “Have you had any sores or infections around the vagina?" Pregnancy: + “Have you ever been pregnant?”, "How many times?", "Any miscarriages er abortions?". «If YES “How many times did you abort? In which month/week of your pregnancy? Do you know the reason (s) for the abortion?” * "Have you had any other problems or complications with the pregnancies?” + "How were the births?" Have you had any complications during delivery?". Abdominal pain + “Heve you ever had any pain in your belly?” * IFYES continue with all the questions given under pain in present history Pap smear: + “Have you been getting regular pap smears?", "When did you have the last Pap smear?". Sexual History: * Before taking the history let the patient know that you wouid be asking about her/his sexual history, so you will be posing a transition question. ( Ok Mr. Brown Now I would like to ask few questions about your sexual history, please understand it will be kept confidential between you and Me, ...try to be as honest as possible. Is that ok with you? ) Continue as follows: © “Are you sexually active?” ‘* IF YES “Who Is your sexual partner?", *Do you have any other sexual partners?" or "Do you relate sexually to men, women or both?”, “Are you satisfied with your sexual life?" History Taking Page 11 of 16 If NO enquire the reason, "Do you have any problems in your sexual life?” " "Any loss of Interest in sex?", "Are you able to reach a orgasm?" + "Do you use any means of contraception?” « IF YES “What type of contraception do you use? Do you use it regularly? « For high risk groups like patients who are not using barrier methods of contraception, patients with multiple sexual partners, patients with homosextial history continue with following questions: (Note: most of the time they have this history and so never miss it. + “Have you ever been tested/treated for sexually transmitted diseases?” « “Have you ever been tested for HIV?” Social history: «You need to pose a transition question: ("OK Mr. Brown naw I would like to know about your social habits and personal life style, is that ok with you?”} Appetite: + “How is your appetite?” Diet: * "Can you please tell me about your diet” * "What does your diet mainly consist?” « “Are you on a special diet?" * For peri/postmenopausal wornen ask "Do you take calcium supplements?". Weight + “Has your weight changed anyway?” © IFYES: How much? In what period of time?” Smoking: * "Do you use tobacco? Do you smoke?” History Taking Page 12 of 16 «IF NO “Have you ever smoked in the past?* (Most of the SP’s have past history of smoking) ‘+ IF YES “How many packs do you smoke per day? For how long have you been smoking?” * "Have you ever thought about quitting/attempted to quit?” Alcohol: * "Do you drink any type of alcoholic beverages?" # IFN “Have you ever consumed alcohol in the past?" + IF YES "What type of beverage do you take? How much do you drink per day? "How long have you been drinking?” + Always keep in ming about the CAGE questionnaires for suspected alcohol abuse cases ( Ex upper GI bleeding, Right upper quadrant pain, epigastric pain.) + “Have you ever tried to cut down on alcohol drinking?” + “Have you ever been annoyed by other people for your drinking?" * "Have you ever had guiity feelings about your alcohol drinking?” * "Do you drink alcohol early morning?" Drugs: «= “Are you currently taking any type of over the counter medications? any prescription medications?" » "Have you ever tried any recreational type of drugs?” « IFYES to any of the questions ask "What kind of drugs?”, "How long have you been taking them?”, "Have you ever injected drugs?” Occupation & exposure: ©" Do you work? What type of work do you do?” s it a stressful job?" ( Analyze whether It is mentally / physically stressful Ex: mental- Depression; physical: carpel tunnel syndrome « key board users }. History Taking Page 13 of 16 + "Are you exposed to any health hazards in your work or personal life?" * "Are your work conditions safe?” ‘+ "De your job involve prolonged sun exposure?” ( for a case of rash) » "Do you expose to loud noises at work?" (for a case of hearing loss} Exercise: # "Do you exercise regularly?" Stress: * “Do you have any stresses from your family?" Travel: « "Have you traveled outside the United states in the resent years?", "When?", "“Where?”. Angry Patient: ‘+ "Mr, xyz, you seem to be very angry, could you please tell me why that is so? Is there any way that I can help you.” Uncooperative patient: + "Mr. XYZ I can understand your problem, but to properly understand your problem, I have to do this test. It won't take more than a minute. Tam here to assist you. ok?” Pain in hand: ‘¢ "Does your job involve repetitive hand movements like key board operation." (Carpal tunnel syndrome). Insect bite: ‘* "Do you remember being bitten by any insects like ticks and mosquitoes?" for any rash case) Trauma patient: ‘+ Some times you may get a patient with trauma or robbery etc. They wil act like History Taking Page 14 of 16 any thing and you can see all the bruises in different colors with good painting. ‘The main issue with these patients is don't repeat the painful maneuvers. Some times they may werry about their social situation (money). For example @ chest trauma patient from robbery who have signs and symptoms of hemothorax don't want to get chest -X -ray because he doesn't have insurance. So in those cases explain like this "We have a social worker and he/she will find out financial help for you." ‘+ In the USA almost every hospital will have social worker to deal this kind of problems. Over talkative patient + Some times the patient may talk endlessty and irrelevant topics. So in those cases respond like this "Excuse me Mr. xyz, sorry to interrupt you. I know these things have really been bothering you. However I need te focus completely on you (or on your present situation). + Some patients will respond nermaily but some patients will say "Are you interrupting me?". Don't worry they have been told to act like that. Say the same thing again and say sorry once again. General: « Ifyou have to say * I don't know " say “I don't know yet Finally there are 2 popular mnemonics for history taking: LIQOR AAA {LIQOR Is Associated with Alcoholic Anonymous) especially if the chief complaint is a pain. L- Location © I- Intensity Q Quantity +» © - Origin & Duration & Frequency * R= Radiation History Taking Page 15 of 16 «A- Aggravating Factors « A- Alleviating Or Relieving Factors + A- Associated Problems The other mnemanic used for the same purpose is "O PQ RS T" » Onset Provocation/Palliation Quality Radiation * Site Temporal profile The following is very good for past history for all cases. PAM HUGS FOSS” «Previous episodes of chief complaints. « Allergic history «Medical problems in the past ‘+ Hospitalization (Trauma, surgery...) ¢ Urinary complaints ‘+ Gastro intestinal problems * Sleep + Family History » Obstetric and Gynecological History History Taking Page 16 of 16 * Sexual History * Social History Note: This history taking is for ali cases and in general. We made an effort to explain how to elicit history in general and what needs to be asked for certain issues. You need not ask all these Questions for every case. You have to prioritize what needs to be asked for particular case, That will come only with repeated practice of cases that we have given Examination Page 1 of 21 Examination + ‘The alln here is to teach you how to examine a patient in the examination room, ‘+ Aways wash your hands with scap before the physicai examination, don't waste your time while washing your bands, just turn your head towards patient while washing ang maintain conversation. ‘+ Never examine through the gown, but at the same time try to drape (cover) the body as much as you can. Expose only the necessary part. «+ Don’t hesitate to examine heart in a female patient, leaving the bra on, you can ask her to lift her breast for MI ( Point of maximum impulse) and auscultation. ‘+ I |s permissible to place stethoscope down between the breasts to continue listening. Don't do breast examination +The dictum is to inform patient the examination procedure before you examine, Ex: Start like this: You: * Ok Mr. Samson new 1 would like to do physical examination on you, for that I need to untie your gown, is that ok with you?” ‘SP: “Yes its ok” You: “Thank you” + If the patient resists you, then you have to explain the importance of examination to understand the disease process, something like this: + You: "Mr. Samson I can understand your feelings but to clearly understand the disease 1 have to do this examination, it wont take more than a minute don't worry 1 am here to assist you. I will tell you whatever I am going to do.” + Some physical findings can be expressed in SP's face (faked). So be careful and always observe SP's face while examining. Sometimes you may get 2 SP with hypertrophic tonsils as a case of sore throat. + Always do a focused, relevant examination. You wont have much time to de everything. This will come oniy with repeated practice. Respiratory Examination + Ideally the patient should be sitting on the end of an exam table. Inspection: You: "Navr I would like to examine your chest, Is that OK with you?" Examination Page 2 of 21 + Observe the rate, rhythm, depth, and effort of breathing + Usten for obvious abnormal sounds with breathing such as wheezes + Observe for retractions and use of accessory muscles (sternocleidomastoids, abdominals) + Observe the chest for asymmetry, deformity, or increased anterlor-posterior (AP) diameter. Palpation: ‘You: "Nov 1 would like to palpate your chest. Let me knew if it hurts you" + Look for tracheal deviation + Identify any areas of tenderness or deformity by palpating the ribs and sternum, + Assess expansion and symmetry of the chest by placing your hands on the patient's back, thumbs together at the midline, and ask them to breathe deeply. + Check for tactile vocal fremitus, Look for PMI (point of maximum impulse). Percussion: ‘You: "Now I am going to tap on your lungs to look for any flurd ar excess air ‘+ Percuss both anterior and posterior chest from side to side then from top te bottom. Interpretations: IF dull- Pleural effusion or Lobar pneumonia + Hyperresonant - Preumothorax or Emphysema Auscultation: You: "Now I would like to listen to your lungs, can you take some quick nice deep deeps breath for me" ‘+ Use the diephragm of the stethoscope, + Auscultate anterior, posterior and lateral sicies of the chest {rom side to side then fram tap to bottom, Tactile Fremitus ‘You: “Can you please say "ninety-nine" repeatedly for me" ‘+ Palpate with your lateral border of palm. + Increased tactile fremitus suggests consolidation of lung, Examination Bronchophony: You: "Can you please say “ninety-nine, ninety-nine” repeatedly for me?" «This time instead of palpation do auscultation. Auscultate several symmetrical areas aver each lung + Tne sounds you hear should be muffled and indistinct. Louder, clearer sounds are called branchophony {indicates consolidation) Whispering pectorioguy: : "Can you please whisper “ninety-nine, ninety-nine" repeatedly for me?” + Auscultate several symmetrical areas over each lung. + Innormal people you should hear only faint sounds or nothing at all, If you hear the sounds clearly this is referred to as whispering pectorlloquy (Indicates consolidation) Special Points: + Never percuss or auscultate on scapula + Let the Patient fully exhale or inhale before auscultating (shifting to) other area. «For every case of respiratory system don’t forget to auscultate heart and checking for PML Cardiovascular System: + Always examine from patients right side + Always examine in 3 positions. You: “OK Mr. Samson now f would like te examine your heart in sitting position then lying dawn and again in sitting position.” Wile the patient isin sitting postion You: “I need to check your pulses.” cheek for pulses: Note whether the pulse Is regular or irregular + Observe for carotid pulse. + Auscultate for carotid bruits (a sign of arterial narrowing and risk of 2 stroke) for middle aged or elderty patients. + check for dependent edema Examination Page 4 of 21 Don't measure blood pressure unless otherwise asked (they usually give the B.P). ‘Then ask the patient te lie down, You: "Lneed to check your heart, could you please lie doven for me? rll pull out the leg extension to make it @ little more comfortable for you" Look for Jugular Venous Distention (J¥D). Position the patient supine with the head of the tabie elevated to 30 degrees. Inspect for precordial movement. ‘Then palpate for precordial activity ‘You may feel thrils er exaggerated ventricular impulses. palpate for the point of maximal impulse (PMT or apical pulse) Note the location, size, and quality of the Impulse. Auscultation: You: “Now f would like listen to your heart sounds” Position the patient supine with the head of the table slightly elevated. record $1, $2, ($3), (S4), 88 well as the grade and configuration of any murmurs ("two over sx" or "2/6", "pan systolic” or "crescendo"). Listen with the stethoscope diaphragm at the right 2nd intercostal space near the sternum (20r1/¢ area). Listen at the left 2nd intercostal seace near the sternum (puimonary area). Listen at the left 3rd, 4th, and Sth inter costal spaces near the sternum (tricuspid area). Listen at the apex (mitral area). Then ask the patient to turn to left side and then listen mitral area Record the rate, rhythm, and any exire sounds you Hea. Ask the pa nt to sit up again. Again auscultate all 4 areas ‘Ask him to lean forward and listen at the base of heart. Examination of the Abdomen: Examination Page § of 21 «+ The patient shoule be lying supine on the exam table and aporopriately draped, + Watch the patient's face for signs of discomfort during the examination. + Consider the inquinat/rectal examination in males and pelvicfrectal examination in fernales. «+ Don't do these examinations but tell the SP that you want to do them later. Inspection ‘You: "am looking at your belly for eny gross abnormalities” + Look for scars, swellings, and hernias. + Look for any visible peristalsis or pulsations + Noxe the abdominal contour (Flat, scaphoid, or any abdominal distention). Auscultation: You: " Now I would like to listen to your bowel sounds” «+ Ibis very important to auscultate prior to palpation 1. Listen for bowel sounds. 2. Listen for bruits aver the renal arteries, ilac arteries, and aorta Percussion: You: "Wow 1 would like to tap on your belly" «+ Percuss in all four quadrants. + Percuss the liver span 1. Pereuss downward from the chest in the right midclavicular line until you detect the top edge of liver dullness, Percuss upward fram the abdomen in the same line until you detect the bottom edge of liver dutiness. 3. The distance between the points is the liver span # Check for splenic dullness |. Percuss the lowest costal interspace in the left anterior axillary line. This area is normally tympanic. Examination Page 6 of 21 2, Ask the patient to take a deep breath and percuss this area again. Duilness in this are may be 2 sign of splenic enlargement. Palpation: You: “Now I would like to press on your belly first lightly then deeply. Please let me know if it hurts” + Start with light (superficial) palpation, This is to look for any areas of tenderness. Look at the patient's face not on the abdomen. The most sensitive indicator of tenderness is the patient's facial expression. Also look for guarding (Voluntary or involuntary). + Proceed to deep palpation after light palpation Palpate the Liver: + Place your fingers just below the right costal margin and press firmly then ask the patient to take a deep breath. Palpate the aorta: If you suspect any aneurysms: Palpate the spleen: Special Tests: Rebound Tenderness: + This is a test for peritoneal irritation. + Tall the patient what you are about to do. "Now I would like to press in and release, please let me know if it hurts when I press in or let go.” + Press deeply on the abdomen with your hand then after a moment, quickly release pressure, «If t hurts more when you release, the patient has rebound tenderness. Costovertebral angle Tenderness (CVA): + CVA tenderness is often associated with renal disease. «Tell the patient what you are about to do, "Now I would like to give few thumps on your back, Please let me knove if it hurts.” 1. Ask the patient to sit up. 2. Use the heel of your closed fist to strike the patient firmly aver the costovertebral angles. 3. Compare the left and right sides, Shifting dullness. Examination Page Tof 21 + Can be done for Nuld in the abdomen. ( Usually not required for CSA) Psoas sign: ‘+ This is a test for appendicitis. First ask the Patient to turn op to the Lt side. Then extend the patients ght leg. This will cause stretching of the Psoas muscle and will cause pain if it's irritated by inflamed appendix. SP can act exactly like a real patient, se watch his face. obturator sign + This is also a test for appendicitis |. Raige the patient's right eg with the knee flexee 2. Rotate the leg internally st the hip. 3. Increased abdominal pain indicates a positive obturator sign. Murphy's sign: You: “Naw I would like te press on your belly, please take @ breath and please let me know if it hurts.” + This is for suspected cholecystitis. Hook the finger of your right hand underneath the right costal margin at the point where the lateral border of the rectus muscle intersects with the costal margin. Then ask the Patient to take deep breath. If he complains of pain then the test is +ve. Neurologic Exarnination: + Always consider left to right symmetry. + Organize your thinking into seven categories: 1. Mental Status Cranial Nerves 4. Coordination and Gait Motor system 5. Reflexes 6 Sensory system 7. Special Tests Mental Status: + Level of Consciousness: You: "Hello Mr. Swanson, can you hear me If you hear me can you smile for Examination Page 8 of 21 Orientation in time, place and person: You: "Where are we now?, what is the date today?, can you please tell me your full name?”, ‘Assess Judgment: You: "What would you do if you found a self addressed envelop on the street You have to test for memories of recent, remote and registration., Test memory: Recent remote registration. Simple way is "Mr. Swanson, I am going to tell you 3 wards. I want you to remember them and repeat ther immediately. 1 will ask you to repeat the same words after 5 minutes again, Ok, Am I clear?" Tell him "Pen, Car and chair’. Ask again at the end of mental status. For fong term memory you can ask like "What happened on September 11th?", Abstraction: You: “Can you please spell the word "WORLD" backwards for me?" Three Object Command: "Mr. Swanson, | em going to wrte a sentence here, please read the sentence and do as it says, Ok!". You write "Close your eyes" on a paper, Some people do serial 7's test but it is time consuming, so we don't acvise that. If you do above 6 things that is mare than sufficient for this exam Cranial nerves ‘Observe for any gross abnormalities like Prosis (111) Facial droop (or) asymmetry (vit) Abnormal articulation of words (¥, VII, X, XI). Abnormal eye position (IIL, 1v, VI} Abnormal or asymmetrical Pupils (IL, 1. Olfactory: Optic: Usually not tested for this exam, ‘You: "Now I need to examine your eyes, can you please count my fingers". Examine the optic fund ‘Test visual acuity by using Snellen’s chart or simply By using your fingers. Screen visual fields by confrontation Examination Page 9 of 21 + Stand two feet in front of the patient and have them look inte your eyes. Hold your hands about one foot away from the patient’s ears, and wiggle a finger on one hand. Ask the patient to indicate which side they see the finger move, Repeat two or three times te test both temporal fields. If an abnormality 's suspected, test the four quedrants af each eye while covering the other with the patients hand ‘+ Test pupillary reactions to Light. |. Dim the room lights as necessary. Ask the patient te look into the distance. Shine a bright light obliquely into each pupil in turn 4. Look for both the direct (same eye) and consensual (other eye} reactions: 5. Record pupil size and any esymmetry or irregularity Oculomotor: + Observe for ptosis. + Test extra acular movements + Ask the patient to Follow your finger with eneir eyes erthout moving their head. + Check gaze in the six cardinal directions using a cross or "H” pattern. + Pause during upward and lateral gaze to check for nystagmus. + Check convergence by moving your finger towtard the bridge of the patient's nose. + Test puptlary reactions to light Trochlear + Test extra ocular movements (Inward ane downward movement) Trigeminal + Test motor strength. + Ask the patient to clench his/her teeth, «+ Paipate the temporal and masseter muscles as they do this. + Test the 3 divisions for pain sensation. Examination Page 10 0 + Exotain what you intend to do. + Use a suitable sharp object to test the forehead, cheeks, and jaw on both sides. + Substitute a blunt object occasionally and ask the patient to regort "sharp" or “cull.” + Test the corneal reflex (Usually not required for this exarn). Abducens: + Test extra acular movements (Lateral movement} Facial + Observe for any facial droop. + Ask patient to do the following, note any lag, weakness, or asymmetry: + Raise eyebrows ("Can you please raise your eyebrows for me?"). Close both eyes to resistance (" will try to open your eyes, con't let me open."). + Sinile { "Can you please smile for me?). + Frown (*Can you please frown for me?*). Show teeth ( ‘an you please show your teeth for me" ‘+ Test the ‘corneal reflex’ (Usually not required for this examination). Acoustic: + Screen hearing «Test the hearing by making simple noise with Angers, + If abnormal, then do Weber and Rinne tests. + Test for lateralization (Weber) + Use 2 512 iz tuning fork, + Place the base of the vibrating tuning fork firmly on top of the patient's head. + Ask the patient where the sound appears to be coming from (normally in the midline) + Compare air and bone conduction (Rinne) + Use a 512 Hz tuning fork. Examination Page 11 of 21 + Place the base of the vibrating tuning fork against the mastoid bone behind the ear. + When the patient no longer hears the sound, hold the end of the fork near the patient's ar (air conduction is normally greater than bone conduction). Vestibular function Is not normalty tested. Glossopharyngeal: Usually not required for this examination, Vagus Listen to the patient's voice, is it hearse or nasal? + Ask the patient to swallow. + Ask the patient to say "AR" + Watch the movements of the soft palate and the pharynx. + Test ‘gag reflex’ (Unconscious/Uncaoperative Patient) (Usually not required for this examination) Accessory: + From behing, ook for atrophy or asymmetry af the trapezius muscles. + Ask the patient to shrug his shoulders against resistance ("Can you please shrug your shoulders for mer"). + Ask the patient to turn his head against resistance. Watch and palpate the sternomastoid muscle on the opposite side ("Can you please tura your head against my head?"), Hypogiossal: + Listen to the articulation of the patient's words. + Ask patient to stick out the tongue then ask him to move tongue from side to side ("Can you please stick out your tongue? and move it side to side") Motor: Observation: + Involuntary movements + Muscle symmetry and atrophy. Always compare both (right and left) sides. Check is there any difference between proximal and distal muscle groups. + Gait Examination Page 12 of 21 Muscle tone You: "Now I would like to check your muscle tone, please relax your muscles, OK" + Flex and extend the patient's fingers, wrist, and eloow. + Flex and extend patient's ankle and knee. + There is normaily a small, continuous resistance to passive movement. + Observe for decreased (faccid) or increased (rigid/spastic) tone Muscle Strength: Yo "Now 1 would like to check your muscle strengthypower’ ‘+ Test strength by having the patient move against your resistance. + Always compare one side to the other. + Grade strenath on a scale from 0 to 5 “out of five (o/5][No muscle movement at all i:75|ust flicker of movement. [Movement at the joint, but not against [2/5 Jgravity 'ay5)Mevement against gravity, but not lagainst added resistance [Movement against resistance, but less “than normal 5/5|[Normal strengtn ‘Test the following, + Spine ("Now I woul! like to check the flexi touch your toes”) y of your spine, please lean over at the waist, and try to Upper limb: + Flexion at the elbow (“can you please pull in”). + Extension at the elbow (“can you please push out”), + Flexion and Extension at the wrist (“can you please pull up and push down”) + Squeeze two of your fingers as hard es possible ("can you please squeeze my finger as tightly as you cen"). + Finger abduction {uinar nerve) ("can you please spread out your fingers”). Examination Page 13 of 21 + Oppositian of the thumb (median nerve). Lower limb: + Flexion at the hip (iliopsoas): ("can you please pullin”) ‘+ Adduction at the {adductors): (“can you get your thighs closer to each other”) ‘+ Abduction at the hips (gluteus medius and minimus): ("can you please move out your thighs one fram the other’) + Extension at the hips (gluteus meximus): ("can you please push out your thighs”) + Extension at the knee (quadriceps): ("can you please kick out") + Flexion at the knee (hamstrings): (“can you please pull in") + Dorsiflexion at the ankle: (*can you please pull in”) + Plantar flexion (St): (“can you please push down") Coordination and Gait: Cerebellar Function Tests: + Coordination and gait ("Now 1 would like to check your balance and movements”) + Rapid alternating movements i.e, ask him ta do rapid supination and pronation of hands. + Point-to-Point Movements: “Can you please touch your finger to my finger and then to your nose”. Ask the patient to touch your index finger and their nase altertiately severat times, Move your finger about as the patient performs this task. + Ask the patient to place one heel on the opposite knee and run it down the shin to the big toe. Repeat with the patient's eyes closed. If you do the above test you don't need to do this one, Rombera’s' test: You: “Can you please stand up for me and get your feet together. Can you please close your eyes, don’t worry Twill assist you if you fall” + Be prepared to catch the patient if they are unstable, + Ask the patient to stand with the feet together and eyes closed for 5-10 seconds without support. 1+ The test Is said to be positive if the patient becomes unstable (its a test for sensory ataxia not for cerebellar ataxia) Gat: Examination Page 14 0f 21 ‘You: “Can you please walk few steps for me”. + Ask the patient te rise fram a sitting position, walk across the room, turn and come back ( Get up and 90 test) + Ask the patient to walk in a straight line: “Can you please walk in a straight line for me” Reflexes © Miss CSA, now I would like to check your jerks. First, let me begin with your arms, Then, let me check your legs. Please relax your muscles" Deep Tendon Reflexes: + The patient must be relaxed and positioned properly. + Reflexes can be reinforced by asking the patient to perform isometric contraction of ether muscles (clenched teeth}: (“Can you please clench your teeth for a white”) + Reflexes should be graded on a0 to 4 "plus" scale: (o_J[absent IHypoactive (diministed) 2+ |[Normar’ lExaggeratea without Iclonus [Exaggerated with Iclonus a a+ las + Biceps (C5, C6). + Triceps (C6, 7). + Brachioradialis (C5, C5) + Abdominal (18, 79, T10, T11, T12}. + Knee (L2, L3, La). + Ankle (S1, $2). Conus: «Ifthe reflexes are exaggerated, test for ankle clorus. + Plantar response (Babinski): Stroke with @ blunt ebject along the lateral border of the feet. Extension of the big toe with fanning of fingers is abnormal. This \s referred to as a positive Babinsk', Examination Page 15 of 21 Sensory: + You: "Now I would like to check your sensory perception in different areas of the body" General: + Explain each test before you do it. + Patient's eyes should be closed during the testing. + Compare symmetrical areas on the right and fen. + Also compare distal verses proximal Vibration: + Use a low-pitched tuning fork (128H2). + Test with a nen-vibrating tuning fork first to ensure that the patient is responding to the correct stimulus. ‘+ Place the stem of the fork over the distal Interphalangeal joint of the patient's index fingers and big toes. + Ask the patient to tell you if he feels the vibrations, + IF vibration sense is impaired proceed proximally 1. Upper limb: Fram wrists, elbows ete. Lower limb: From medial malieoll, patella, anterior superior iliac spine. Position Sense: + Grasp the patient's big toe and show the patient "up" and “down.” «+ With the patient's eyes closed ask the patient to identify the direction you move the tee + If position sense is impaired move proximally to test other joints, + Test the fingers in a similar fashion, «Tf vibration, position sense, and subjective ight touch are normal in the fingers and toes you may assume the rest of this exam will be normal Pain: You: "Please understand this is sharp and this is dull, now close your eyes and let me know which is what” Examination Page 16 of 21 + Use a suitable objects to test “sharp” and “dull” sensations, ‘Test the following areas: + Shoulders (C4), 4+ Inner and outer aspects of the forearms (C6 and T3). + Thumbs and little fingers (C6 and C8). ‘+ Front of both thighs (L2). + Medial and lateral aspect of both calves (L4 and L5). + Little toes ($1) Femperature: + Often omitted if pain sensation is normal. Light touch: + Use a fine wisp of cotton or your fingers to touch the skin lightly. + Ask the patient to respond whenever a touch is felt, + Test the following areas: Shoulders (C4) + Inner and outer aspects af the forearms (C6 and Tt) + Thumbs and little fingers (C6 and C8) + Front of both thighs (L2) + Medial and lateral aspect of both calves (14 and LS) + Little toes (S1) Disenmination: ‘+ Since these tests are dependent on touch and position sense, they cannot be performed when the tests above are cleariy abnormal. + Two point discrimination - Using 2 pins (pointed edges) touch finger pad in 2 places simultaneously. Alternate with a one point touch, The distance between the 2 points should be <5 mm on the finger pads. Examination Special tests: Kerning's Sign: Here the patient lies on back. Flex bath knee and hip then try to extend the knee only. In meningitis, patient will complain of pain or resistance to knee extension. Brudzinsk'’s Sign: When you flex the neck, there may be a flexion of the patient hips and knees. This is, also seen in meningitis patients. Neck stiffness: Have the patient lie on back, place your hand behind the patients neck and flex it anti the chin touches the sternum. In meningitis he will complains of neck pain and you will notice resistance to motion. Examination of the Head and Neck Head: Look for scars, tumps, rashes, hair loss, or other lesions Look for facial asymmetry, involuntary movements, or edema. Palpate to identify any areas of tendemess or deformity. "Now I need to examine your eyes" Test the visual acuity by using Snellen’s chart. Inspection: ‘+ Ask the patient to look up and pull down both lover eyelids to inspect the conjunctiva and sclera. Note any discoloration, redness, discharge, or lesions, Note any deformity/lesions of the iris and cornea, Extra ocular muscles: You: “Iwill shine this light on your eyes, please iook at some objects”, Corneal Reflections: + Shine a light from directly in front of the patient, + The comeal reflections should be centered over the pupils, + Asymmetry suggests extra ocular muscle pathology. + Test for extra ocular movements + Test visual felas Page 17 of 21 Examination Page 18 of 21 + Look pupillary reactions to light Ophthalmescopie exam + Darken the room as much as possibie. «+ Adjust the ophthalmoscope se that the light is no brighter than necessary «Use your left hand and left eye te examine the patient's left eve. Use your right hand and right eye to examine the patient's right eye. + Ask the patient to look at a point on the wall «Examine retina and fundus and note any abnormalities Ears: + Inspect the auricles and move them around gently. Ask the patient if this 's painful + Palpate the mastoid process for tenderness. «Hold the otoscope then pull the ear upwards and backwards to straighten the canal, 1s Insert the otoscope. Inspect the ear canal and middle ear structures and note any redness, drainage, or deformity. Nose: Inspect the nose + Tilt the patient's head back slightly. «Insert the otoscope into the nostril try to avoid contact with the septum, Inspect the visible nasal structures and note any swelling, redness, drainage, or deformity: Throat: You: “I need to look inside your mouth for any ulcers or dental problems” ‘+ Ask the patient to open their mouth. + Note any ulcers, white patches (leuconlakia), or other lesions 4s Inspect the posterior oropharynx by depressing the tongue with tongue depressor and asking the patient to say “Ah.” Note any tonsillar enlargement, redness, or discharge: Neck: Examination Page 19 of 21 ‘Inspect the neck for asymmetry, scars, or other lesions, + Palpate the neck to detect areas of tenderness, deformity, or masses. Lymph Nodes: You: “Now T would like to check for any swollen glands, let me press on this area” + Systematically palpate the various lymph node groups, + Check superficial cervieal, supra clavicular, and the deep cervical lymph nodes. The deep cervical chain of lymph nodes lies below the sternomastoid and cannct be palpated without getting underneath the muscle, [nfarm the patient that this procedure will cause some discomfort. Hock your fingers under the edge of the sternomastoid muscle. Ask the patient to bend thelr neck toward the side you are examining. Move the muscle backward and palpate for the deep nodes undemeath. ‘+ Note the size and location of any palpable nodes and whether they were soft or hard, non-tencer or tender, and mobile or fixed Thyroid gtand: You: "Let me check you thyroid, I am going to press very gently gently on this area” ‘+ Inspect the neck locking for the thyroid gland. + Note the size, symmetry, and position of the lobes, as well as the presence of any nodules. The normal gland (is often not palpable. Special Tests: Facial tenderness: You: "Now T would like to check your sinuses 1 am going to press on your forehead and sinuses, please let me know iF IU hurts” + Ask the patient to tell you if these maneuvers cause excessive discomfort or pain. + Press upward under bath eyebrows with your thumbs. + Press upward under both maxillas with your thumbs + Excessive discomntort on one side or significant pain suggests sinusitis, Sinus transillumination: + Darken the room as much as possible, + Place a bright point light source on the maxilla, Examination Page 20 of 21 1s Ask the patient to open his mouth and look for an orange glow on the hard palate, 4 decreased or absent glow suggests that the sinus is fled with something ether than alr. Temporomandioutar joint: You: "Now I need to examine your jaw joint” + Place the tias of your index fingers directly in front of the tragus of each ear then ask the patient to ‘open and close their mouth, + Note any decreased range of motion, tenderness, or swelling, Musculoskeletal System: Inspection + Inspect the area. + Always compare with the opposite area. 1+ Always compare other joints for any signs of abnormality. Palpation: = Palpate the area for warmth, and took for any swelling. + Palpate for tenderness and crepitus, + Check range of motion. «Always check the neurological status (motor, sensory, reflexes.) and vascular status of thar area, Special situations: Knee: «Always do Drawers test and Mc.Murray's test if patient has 2 H/O trauma to the knee, + Click here for Drawers test, + Click here for Me-Murray's test. Shoulders: “« Range of motion is crucial; ask him to touch the opposite shoulder: vavist: + Cheek neurological (dermatomes) status. Examination Page 21 of 21 Spine: + Check the flexibility, gait and reflexes Leg: ‘+ Homan’s sign ~ this is usually dene for suspected deep vein thrombosis, Forceful Gorsiflexion of the foot will cause pain the calf. 5 Page | of 4 Headache 4.27 yr old white female Laura Linas comes to you with the complaint of headache, Her vitals are as follows: BP 120/70 mm of Hig Temp 98.6 F RR 19/min HR B0/min Simulated encounter: Knock the door “Hi, Mrs. Laura? Good Morning, My name is Dr. XYZ. Nice to meet you. How are you doing today? What brought you in today?” (Dec I am having headache.) Can you please tell me something more about your headache? (Doc I am having trouble with the headache) For how long have you been having this headache? {For the past several hours) Do you know how this pain has started? { mean was it all of sudden in onset or gradual? (It started suddenly) Js it a constant or intermittent type of pain? (Its pretty much constant) Can you please show me exactly where the pain is? (Alll over my forehead) Does the pain go anywhere else? Like to your jaw or back of the neck? (No) What were you doing before you noticed the headache? (I was in my office} How do you describe your pain? (It’s a band like sensation) Ina scale of 1 to 10, which number would describe your pain? (1 would say probably 7-8/10) Is there anything that makes your pain better or relieve? (Yes, staying in dark room) Is there anything that makes your pain worse or bad? (Exposure to bright light and moving here and there) Have you had any nausea or vomitings? (Little bit nauseation but not voritings) Have you ever had this type of pain before? (Yes, couple of times 3 months ago) You said you had this pain before, right. When you get these headaches, for how long do they ‘ast? Do you get to know that you are going to have headache? I mean do you feel any blurriness or seeing flashes nefore you get the headcahe? (Asking the premonitory symptoms) (No, 1 don’t know when it’s going to start) Do you have any blurriness or double vision now? Being an episodic headache, do you think is it affecting your daily activities? (No, I am working as usual) ROS (For CSA, review of systems is very focused): 8 Page 2 of 4 ‘Ok let me ask you some other questions very quickly. Bo you have any fever (No}, chills (No}, 0° you have any neck stiffness? (No). Have you had any watering of eyes? Have you had any discharge from the nose? Have you noticed any ear discharge? Have you had any trauma to your head? Have you had any weakness in the arms or legs? Have you noticed any sensory changes ike tingling or numbness in the hands or legs? Ask each question after a brief pause. De you have any problem with your bowel movements? Has there been any change in appetite? Have you lost or gained weight lately? Do you have any urinary complaints? PMH: Do you have any other medical problems? (No) Bo you have any H/0 high blood pressure? (No) Have you ever been hospitalised before? all Are you allergic to anything? Medications: Are you taking any prescription of over the counter medications? Have you ever taken recreational drugs? (No) Do you use any hormonal contraception? (No) FH: Can you please tell me something about the health of your family members? (They are fine}. oes anyone in the family nave headaches? (Yes, actually my sister has migraine, she wanted me to have a checkup with the doctor, and she thinks that it might be migraine. What do you think? (Ms, Lines, from whatever you told upto now, there is a possibility that it can be migraine headache. But Teed to ask few more questions and physically examine you. It will help me to tell you something more certain. Is it Ok?} (Sure doc) si What do you do for living? Or where do you work? Do you feel any stress at work or home? Bo you smoke? (No), Have you ever smoked? (No) Do you take alcohol? How much and how often? Are you sexually active? Are there any problems in your sexual life? Physical examination: Lsten for bruit at neck Palpate the head, neck, and shoulder regions. Check temporal arteries in elderly patients. Examine the spine and neck muscles, ‘A functional neurological examination including getting up from a seated position without any support, walking on tiptees and heels, cranial nerve examination, fundoscopy and otoscony, tandem galt and Romberg test, check for symmetry on motor, sensory, refiex and cerebellar (coordination) tests. Counseling: Tell her the possible diagnosis and neeed for further workup Call the patient by name Acknowledge the discomfort of the patient Differential Diagnosi Migraine Ss Page 3 of 4 Cluster headache Tension headache Subarachnoid hemorrhage/CVA Sinusitis Brain tumour Meningitis/Encephalitis/Infections ‘Temporal arteritis (In the eidelry pt) Refractive erros (If they give any positive history) Medications/Drugs Investigations: CBC with differential ESR Sinus X-ray CT head with out contrast LP (Not in this patient; Pt with fever, confusion and appears sick, You should also need to take blood cultures for suspected meningitis) Short explanation: A number of other disorders may cause headaches, including sinus disease and brain tumors Patients frequently attribute headaches to eyestrain, Evaluating @ patient with new onset headache pain can be challenging. It requires @ systematic approach based upon an understanding of the common headache syndromes. History: Paying attention to danger signs is important since headaches may be the presenting symptom of a space-occupying mass or vascular lesion, infection, metabolic disturbance, or a systemic problem. The following features in the history can serve as warning signs of possible serious underlying disease. Severe persistent headache, which reaches maximal intensity within a few seconds or minutes after the onset of pain, warrants aggressive investigation. Subarachnoid hemorrhage, for example, often presents with the abrupt onset of excruciating pain, In contrast, migraine headaches generally begin with moderate pain and then gradually increase to a maximal level over one to two hours. Cluster headache may sometimes be confused with @ serious headache, since the pain from a cluster headache can reach full intensity within minutes, However, cluster headache |s transient (usually fasting less than one to two hours) and is associated with characteristic ipsilateral autonomic signs such as tearing or rhinorrhea, ‘The absence of similar headaches in the past Is another finding that suggests a possible serious disorder. The “first” or "worst" headache of my life is # description that sometimes accompanies an intracranial hemorrhage or central nervous system (CNS) infection. On the other hand, patients suffering from migraine usually have had similar types of headaches in the past. Infection in a non-intracraniai location (such as the lungs or paranasal or masteid sinuses) may serve as a nidus for the development of meningitis or intracranial abscess. Fever is not 2 characteristic of migraine headache; It may, however, follow a subarachnoid hemorrhage by a few days. Any change in mental status, personality, oF fluctuation in the level of consciousness suggests 2 potentially serious abnormality 8 Page 4 of 4 The rapid onset of headache with strenuous exercise, especially when minor trauma has occurred, raises the possibility of carotid artery dissection or intracranial hemorrhage. Head pain that spreads into the lower neck and between the shoulders may indicate meningeal irritation due to either infection or subarachnoid blood; it is not typical of a benign process. Chronic nasal stuffiness or chronic respiratory infection suggests a diagnosis of sinusitis Impaired vision or seeing "holes" around light suggest the presence of glaucoma. Visual field defects suggest the presence of a compressed optic pathway (eg, due to a pituitary mass). Blurring of vision on forward bending of the head, headaches upon waking early in the morning that improve with sitting up, and double vision or loss of coordination and baiance should raise the Suspicion of raised intracranial pressure; this disorder should also be considered in patients with chronic, daily, progressively worsening headaches associated with chronic nausea. The presence of nausea, vomiting, worsening of headache with changes in bedy position {particularly bending over), an abnormal neurologic examination, and/or a significant change in prior headache pattern suggested that the headache was caused by @ tumor. Sudden, severe, unilateral vision loss suggests the presence of optic neuritis. Headache, fatigue, generalized aches and pain, and night sweats in subjects age 55 or older suggest the presence of temporal arteritis, Intermittent headaches with high blood pressure are suggestive of pheochromocytoma. Source/Reference: UpToDate 40 Page 1 of 5 40-year-old woman who has become withdrawn ‘This case scenario will be presented ina different format in that the answers given by the SP will be given in parentheses Inthe whole patient encounter the SP will be ina most disinterested mood and will talk in a feeble voice ‘+ The doctor should always make eye contact with the patient «The paucity of speech trom the patient should not frustrate the doctor Simulated encounter «+ Hello Mrs, Jones. My name is Dr. Smith, | am here to ask you 2 few questions and do a physival on you, Is that ok with you. (The SP will nod feebly) What brings you in today (I don’t know Doctor. I feel a bit down) ince when have you been feeling this way (3 months. Maybe) «© Mrs. Jones: do you th nk that something has brought this on.¢ She remains silent) Mrs, Jones; | know that you are ita lot of emotional stress. Are you willing to share your problems with me? (The patient Jooks at the doctor and then away) » Do you think any thing bas brought this on (I don’t think so) + Do you have anybody to talk to you when you feel down (J have an aunt, She stays far away) # Mrs, Jones.... How has your appetite been of late (1 don’t fee! like eating) Mrs. Jones... Have you fost or gained any weight recently (my weight decreased by 7 Ibs over the past one month) How has your sleep heen? (I am getting up carly in the morning) Do you feel guilty for any reason (1 don’t think | am being a zood mom for my children) «Do you teel any tiredness (F don’t toel that I have uny energy at all. 1 just don't want te get out of the couch) 40 Page 2 of 5 + Mrs. Jones... Please describe your daily routing for me (I am a house wite. 1 do the housework and cook for my children. Thats about it.) + Can you tell me if'you have any hobbies and imerests (I play violin and sing in the eboir on Sundays; but now don’t feel like doing that anymore) «© Do you feel that you are losing your power of concentration (mmm | don"t know) = Are you forgetting things more offen (Yeah. 1 am forgetting to pay the bills ete) * Whar do you enjoy most (Spending time with my children) «Do you still enjoy it ( don’t fee! like I want to be with any body right now) «Mrs. Jones... Have you ever felt like ending your life because you felt that it wasn’t worth living? (Yeah — a couple af times) + Do you have any ideas on how to do it (No) + Do you have guns or pills at home (Yeah. | have 32 at home) + Do you feel cold when others don't (no) ‘* Ave you losing any hair (na) « Have you ever listened to or scen anything that others don't ( 0) # Please do an MMSE at this juncture « Do you think that some thing is going wrong, with you (Yeah) «Are you willing to get help from a counselor (1 don’t know) « {can get you in touch with one. (Ifyou Feel that would help me.} Now I need to ask you a few questions about your health in the past. Ts that ok with you ¢Yeah) «Have you ever been hospitalized in the past (No exeept for the deliveries of my children) ‘¢ Have you ever felt like this in the past (no) 40 Page 3 of § Are you on any medications (no) Do you have any allergies of any kind (none that f know of) * Do you have any problems with your urination (ne) Do you have any problems with your digestion or your bowels (1 have been constipated of late) «Have you ever had any shoriness of breath (no) « Have you ever had any chest pain (20) © Have you had cough that just wouldn't go away? (no} Now I need to ask you a few personal questions. Please do not feel embarrassed. Everything you say will be kept confidential + Are you sexually active (yes) How many sexval partners do you have-A couple of them Your sexual preference is ...(imales } + Do your sexual pariners use condoms (Yes they do) ‘+ Have you ever been diagnosed or treated for an STD (no) # Have you ever been tested for HIV (no) Now [ved to ask you a few questions about your family so that L can get a clearer picture of your health + Are you married (J was. Got a divorce 10 years ago) ‘+ Are your parents living (no. They died ef old age) + Has any one in your family been diagnosed with x psychiatrit disorder (no) «How many children do you have(2. The elder one is 14. The younger one is 11) Now I need to ask you a few questions about your lifestyle. 40 Page 4 of 5 + Do you use tobacco (no) Do you drink any type of aleoholic beverages? (Yeah. | have 2 shots of seoteh on the ro doing so for the past 10 years.) Do you use any recreational drugs (no) + Do you drink coffee (Yeah. One cup eve morning) Now I need to do a physical on you, Please excuse me fr a few seconds while I wash my hands + Lam going to check your thyroid gland (Not paipuble) + Lam going to check your reflexes (24) + [am going to check your pulse now « Lam going to listen to your beart and kungs now, ‘Thank you Mrs. Jones for your cooperation, « Lam going to sit down and tell you what | think so far + [feel that you are having an episode of depression though | wouldn't definitely say it before the results of some tests that I am going to run on you + Tam going 10 otder a blood test on you to find out if you have any problems with your thysoid. After the results come in T would sit with you again and then we can discuss how we can make you feel better. # Is there anything else that | ean do for you? Differential Diagnosis + Depression + Hypothyroidism # Occult carcinoma Workup 40 « 13 T4TSH + CBC with dilferemial + Urine and serum toxivology sereen Page 5 of 8 Please evaluate Mrs Page | of 3 52 O/F with a Complaint of hot flashes Vital « BP 140/80 «+ Pulse 80/min and regular + RRis 16/min * Temp. 98.8" Communication and Data gathering + Hello Mrs, Armstrong: Lam Dr. Jones thello Dr). good morning (good morning 132). nice to meet you (nice to meet you Dr) ‘© What brings you in to day (I don’t know Dr: I feel some hot flashes that scem to run through ine) Since when has this started (around 3 months ago Dr) * How often do these hot flashes come (About 10 times daily ) « Do you feel anything else when these Hashes occur (I sweat a lot and [ fee! my heart racing) ‘= Do you know that you are going to got « hot fash beforehand-I mean do you feel it coming even bofore it really starts. (Yes Dr-sometimes I do, [1 some times even disturbs my sleep) How are you feeling during most of the days mein what is your mood like during the last 3 months. {I don’t know Dr. | seem to be dull some times and some times | have these uncontrollable temper episodes and some times I just want to be left alone, I just dont of things. Chis whole thing is driving my husbancl crazy) « Do you feel any burning or pain when urinating (yes I do and | feel that | have to rush to the pathroom some times and [wake up a lot during nights te go to the toilet) = When did you have your last menstrual period Mrs, Armstrong (about & year azo) + Do you have any problems with your bowels (no) + Have you had any thyroid problems in the past. (yes Dr. { had a goiter 10 yrs back but it was Please evaluate Mrs Page 2 of 3 operated and removed) + Do you have any other problems like high blood pressure, diubetes...(10) ‘+ Make eye contact and then say Mrs. Armstrong I am going to ask some sensitive questions, It might be embarrassing to you. but it is in your best interests that we have to go ahead with these questions and please Heel fie «+ How has your sexual life been of late (1 don’t know Dr. I get lot of burning sensation: and 1 generally don’t show much interest because of the pain even though my husband wants to do it) «+ [she supportive (Yeah | guess but he is frustrated with the way | have heen behaving) + Do you have any other problems for which you have taken consultation or medication (Ne Dr ‘This is the first time that | am sick ) + Have any of your relatives becn diagnosed with Breast or uterine eaneer (Yes Dr my sister had one breast removed for breast cancer) + Have you ever had any swollen and painfil Limbs, any blood clots in your legs.? (6) + Have you ever had any pain in the legs or back (for osteoporosis). (no) + Do you smoke (no) «Do you drink any type of alcoholic beverages (no) + Do you have any allergies (no) Do any of your family members have a history of elouing disorders (no) Examination + Do Heart, hing, and abdominal exam very quickly and superficially «© Cheek for musele pain in the back ‘+ Check for hyperactive reflexes ‘Counseling Please evaluate Mrs Page 3 of 3 + Teli the pt that the most probable diagnosis is menopause + Offer help to educate the husband about the possible diagnosis «+ Tell her that she is « candidate for HRT even though ber sister had breast cancer Inform her about the adverse affeets of HRT Offer her estrogen cream for the vagina to ease her dyspareunia and her dysuria Inform her that she needs to supplement Caleium in her diet to reduce the risk of asteaporosis. Inform her of the beneficial effect of udequaie wt bearing exercise, D.D for this Case © Menopause «+ Hyperthyroidism + Occult malignaney © Factitious disorder * Chronie fatigue syndrome Work Up CBC with ESR Thyroid profile Serum FSH and LH « DEXA + Pap smear 8 Page 1 of 3 70 Yr. O/F Complaining of Forgetfulness Vitals ’ + Temperature 98.6 + Pulse 76/min, regular + BP 150/85 + RR 16/min Make a Differential Diagnosis Alzheimer's Disease + Vascular dementia + Normal pressure Hydrocephalus + Vitamin B12 deficiency + Hypothyroidism + Masked depression = Chronic Subdural Hematoma Simulated clinical encounter Tam giving only the questions to ask not the manner in which they should be asked. For that 1 ‘think you will have te read the history gathering section of this site. ‘Hello Mrs. Thomson; Iam Dr, Jones; how are you doing today. If you can hear me please nod. + How are you feeling nowadays? Are you feeling sad or ionely ‘+ Do you find any difficulty while walking + Do you have any problems driving to the grocery store + Do you have any problem eating food, or in making meals for yourself * Do you have any trouble with your toilet habits? + Do have any problems while shopping and housekeeping + Are you finding any difficulty in managing your accounts + Have you noticed any wt loss over the past few months? 8 Page 2 of 3 + Do you have any problems with sleep? + Do you have any spells of dizziness or feeling of 2 pounding heart + Did you notice any cold or heat intolerance + Did you have any medical problems in the past + Do you have somebody to take care of you in case of an emergency or do you want me to get you in touch with @ social organization + Please describe your regular diet for me + Are you having any problems with your memory + Are you able to find your way through your house * Do you smoke «Do you drink any type of alcoholic beverages + Are you sexually active = Have you ever had any sexuaily transmitted diseases «Deo you have any family members who had a hereditary medical condition. Examination * Do the following questionnaire as part of the mental status exam + Mrs, Thomson, Can you tell me your full name + Can you tell me what day it is today + Can you tell me where we are now + Please spell the word WORLD back for me «Please repeat the three objects that I 2 going to tell you right now. I want you to tell them to me right away and then again after some time; so please remember them + Please put your left hand on your right hand and bring both hands towards your chest and then back to their original position. + Doe focused neurological exam = Do the get up and go test + Doa fast heart, lungs exam © Page 3 of 3 ‘+ De not forget to do a fundoscopy Counseling Teli her the diagnostic possibilities ‘workup and prognosis If you suspect that she has Alzheimer's; ask the pts permission and then tell the pt about the importance of structured home environment and the precautions to be taken to avoid falls. Explain to the pt the necessity of being regular in taking her medication. See to it that the pt understands her problem, Ask her about her social support and offer any help D.D for this Case + Alzheimer's Disease * Vascular dementia + Normal pressure Hydrocephalus + Vitamin B12 deficiency ‘+ Hypothyroidism + Masked depression Chronic Subdural Hematoma Work Up CRC with differential CT san of head 14,73 TSH S.Creatinine and . electrolytes Syphilis serology New Page 2 Page 1 of 3 50 Yr, old diabetic male came for medication refill Vitals © BP-135/70 + Pulse- 73 © Rrate-16 + Temp- 98.7 History taking: Please remember that you need to use appropriate transitions sentences. This is quick glance of questions how to proceed, «When were you diagnosed with diabetes? + Are you currently taking any medications for diabetes? (yes, wlyburide) + Are you taking your medications regularly’? + Do you think that your medicines are controlling diabetes eiTectively? * Have you ever taken insulin? + How often do you check your blood sugar? or Are you checking blood sugar regularly according to your previous physician advice? + How has your blood sugars been lately?’ | mean can you please tell me how high /at what range were they? + Do you have any problems that you would like to tell? + How is your vision? Do you think is there any change in vision lately? + Do you feel any abnormal sensations in leys like pins or necdlle prick sensations. any tingl or numbness? + Have you ever had any chest pain? New Page 2 Page 2 01 3 + Do you have any breathing problems? # Are you sexually active? Do you have any problems in sexu intercourse’? How are your bowel habits? ‘+ Do you have any problems with urination? Past History « Are you allergic to anything? ‘+ Have you ever been hospitalized for diabetie complications or for any other reason? ‘Ave you taking any medications besides diabetic drugs? Do you have any other medical problems like high blood pressure? Do you smoke? Do you drink alcohol? Do you exercise regularly? ‘= How is your appetite? Have you lost or gained any weight lately? Examination «+ Look fundus for DM retinopathy ‘Test sensation in fegs. Auscultate for carotid bruits. Palpate precordium for PMI. (Cardiomegaly) Auscultate heart. + Check distal pulses in at least two places ions New Page 2 Page 3 of 3 CBC with differential count Blood glucose Ubatc » BUN and Serum Creatinine + Lipid Profile Key point: Even if the SP is wearing shoe oF socks. please don't forget to take them out to examine the fectt! 20 Yr 20 Yr. O/F Complaining of vaginal bleeding Vitals: + BP-90%60 + Pulse - 90 © Reale - 16 # Tomp - 98.8 Make a Differential Diagaosis in your mind # Regular menses © Abortion + Pregnancy # Fetopie pregnaney + He mole + Trauma. Simulated encounter: Please remember that you need to use appropriate transitions sentences. T) questions how to proceed. + When did the bleeding start? + Was it all of sudden in onset or gradual? Page 1 of 4 is quick glance of + Can you describe more about the bleeding. I mean is it a bright red blood or clotted blood. « Ista pure blood or does it contain any tissue like substance (ie, Molar pregnancy tissue) Has it been a contiauous Mow or spotting? > Grape like 20 Yr Page 2 of 4 + What were you doing when it started? | mean were you sleeping or having sex? +» Do you have any other symptoms that you would like to tell besides bleeding, like abdominal pain, fever, vom + Have you ever been pregnant? + When was your LMP? 1 mean how was the flow? How many pads + Can you describe more about your menstrual do you use per day? How many days? are they regular or irregular? + Have you had any history of abortions? + Have you ever been tested for STDs? Past History # lave you ever had any history of bleeding or hospitalizations for bleeding” Sexual History + Are you married « Hyes + Do yor have any other sexual partners? (yes) Do you use any means of contraception? «no + Do you have any oth: seaual partners? ( « Do you use any means of contraception? «When was your fast sexual contaet? Social h story «Do you smoke? 20 Yr Page 3 of 4 + Do you drink alcohol? = Do you use illivit drugs? (cocaine may cause painless (trimester bleeding) Family history + Do you have family history of any bleeding disorders? + Do you have any family history of multiple abortions Examination ‘+ Look for other sites of bleeding, (nose, gums) + Tell that you want to check for orthostatic hypotension, = Auscultate abdomen = Pero abdomer fir liver span + Palpate abdomen superficially. « Palpate abdomen deeply # Check for rebound tendemess. + Ask (0 do pelvie exam. D.D for this Case + Regular menses + Abortion Pregnancy Eotopie pregnancy HL. mole Work Up 20¥r Page 4 of 4 © Pelvic examination + Pregnancy test © CBC and ESR Iransvaginal ultrasound « TSIL. Serum B-HCG levels: Door way information Page 1 of 5 6 year old Scott comes with left sided shoulder pain, vitals were normal Simulated encounter + Once you see the doorway information all you need to do is just note the name of the pt, Take 15- 30 seconds at door way to make a mental checklist of differential diagnosis of shoulder pain. here is no point in spending more time + First knock the door ‘+ Make comfortable eye contact - empathic ‘« Patient on the table will be in awkward position in pain. Don't change position of the patient. stand in front of the patient. Make two to three feet distance from patient; you adapt to his position «+ We advised to stand instead of sitting « Say hello Mr. xyz nice lo meet you, 1am here to ask you some question's and take your physical (in a slow empathic tone) « Don't shake his hand. .because fe will be supporting b (You will noi get points of shaking and you will loose point of causing pain to patient) painiill hand with the opposite hand. ‘© Patient says“ this pain, I cannot even sleep.” + First ask an open ended question, Mr. Scott can you tell me something more about your pain, [le will answer some aspects of pain like location, quality and some others of LIQOR AAA. make & note in the mind, and don't ask them again, If you are caught asking again tell him that you were Just checking. + Ask all pain questions (LIQOR AAA} plus functional impairment (ask questions about, occupational impairment, sleep. help at home), Remember all three will be there in the eheck list «The patient will respond to all LIQOR AAA questions. Ask specifically whether he took any medications and any relief with them, He will say the pain was becoming better alter he took Ibuprofen. > Always ask the precipitating factor of pain: SP will say that he fell down on the stairway at night Door way information Page 2 of 5 while going down to kitchen to drink water + Then ask about deficits: Do you have tingling, .....«)pause) or numbness (no) + Did you notice any swelling/redness after the fail (no) Do you have pain in any other pari of the body (Yes; little at the palm) Are you able to use your arm (No; because painful) + De you feel any weakness (No, only pain) PMH: ‘The ask PAM HUGS FOSS - Now E need to ask you a few questions about your health in the past. Is that ok with you (Yeah) + Ask the second open ended question "How has your health been until now” * Have you ever had any problems with your shoulder (Yes, Injury to left arm 3 yrs back and had a humerous fracture) Do you have any other medical problems ( Yes | did have acid peptic disease) « Are youallergie to anything ( Yes: I am allergic to penicillin) + Hlave you taken any medications ( Yes only Ibuprofen for pais) Do you have any problems with your digestion or your howels (I have been constipated of late) + Do you have any problems with your urination (no) Now I need to ask you a few questions about your family health, Is that Ok with you? (Yes) # Ave your parents living (no. They died of old age) + Has any one in your family members have medical problems (Pulmonary fibrosis to brother and father) Door way information Page 3 of 5 Now I need (0 ask you a few personal questions, Please do not feel embarrassed, Fverything you say will be kept confidential. + Are you sexually active (No) Now I need to ask you a few questions about your lifestyle + Do you use tobacco (no) + Do you drink any type of alcoholic beverages? (Yeah. | have 2 shots of scoteh an the rocks. Been doing so for the past 10 years.) «Do you use any reercational drugs (no) Here ask the another open ended question for social and occupational history. «You can ask like this tel] me something about your lile at work and home’ (Cannot yo to party today because not able to drive) Examination: * Alter your history....ask him like " All right: thank you for being cooperative... now I would like to take your physical: before going physical is there anything you want to tell me before | start your physical. 1 would be happy fo answer" ( Remember he will tell you exactly like this" Please be gentle with my arm doctor}. ‘this is the time to grab one more point for you. Right away console him like "] know that you are in pain. T will ty to do my exam as gentle as | can. is that sounds good’ You will see the relief on face of the patient and an importance of an open ended question. This question will help you to ask and counsel the patient more efficiently. + ‘Please excuse me fr few seconds while [ wash my hands’: + Always start with local examination i.e. painful shoulder ‘+ Expose tite joint properly while draping th other parts, + Belore inspection say (hat you are looking for redness and swelling, don’t just look...he should know that you are looking. Palpate and compare both joints. «« Palpate... Jor swelling, warmth, and erepitus gently...tell him first that you will be very gentle. Say sorry if he complains of tenderness during the examination. Door way information Page 4 of 6 «Most of the times SP will have tenderness on anterior part of shoulder joint. © Check range of motion (ROM) in abduction, adduction, flexion, extension, internal and external rotation (Obviously SP will have restricted abduction beyond 60 degrees i.e, he will complain of pain after 60 degrees), Always adduct the patient's arm acrass the chest (crossover text). « Check reflexes, pin prick sensations + Check the opposite arm » Cheek hand in detail # Just see the legs very quickly + Listen heart and Jungs in 10 10 15 seconds, ‘Counseling: + Explain the probable diagnosis, follow-up after investigations, and the availability of physiotherapy. Diagnosis + Shoulder dislocation + Shoulder fracture Rotator cuff tear Subucromial bursitis » Ligament sprain Work up 2 CBC + X:ray of shoulder joint two views, including elbow + X-ray hand two views Doar way information Page 5 of © MRI of shoulder + ANA and Rheumatic factors Note: You will get the case exactly similar to this in the real time exam. Please don't ask how come. ‘The important thing that you need to remember from this case is "You have to ask all PAM HUGS FOSS for all the cases no matter what the complaint is because they will have those in the cheek list, They see whether you are asking all the aspects or nol. Believe me "Ask PAM HUGS FOSS for all the eases, you will pass. Doorway information Page | of 5 Doorway information = A.40 yr old white female (Mrs. Kelly) came for obesity evaluation , «Her vitals are BP - 150/90 mm of Hg, HR - 68/min, RR - 16min and temperature - Normal How do you approach this patient? # Knock the door and enter the room with a smiling lace. «+ Hello Mrs, Kelly. this is De. Robert walker, good morning; nice to meet you. (nice w meet you doctor) How are you doing today’! (Good) + Excellent. So what brought you in today? ( You know doctor [ am veally worried about my weight; these days Lam really gaining weight, it makes me worried) Tam glad that you came here for evaluation, We will try 10 work together and try to fix it. ok? (yes doctor , thank you) I know you are concerned about your weight gain: could you please deseribe me little bit more about your problem (I don’t know anything specitic doctor but f am concerned about my weight) [Note: Remember they reveal only few things, They really don’t tell you until you ask specific questions. So you have to start asking relevant questions, Before you ask, make a checklist of problems associated with obesity. Here are the common problems associated with obesity 1. Type TI diabetes 2. Heart disease 3. Stoke 4. Hypertension Osteoarthritis 6. Sleep apnea Doorway information Page 2 of 5 7. Breathing problems 8. High blood chotesterol 9. Gall bladder disease 10. Increased incidence of cancer like endometrial, colon, postmenopausal breast cancer etc. 11, Increased surgical risk 12. Menstrual irregularities 13. Stress incontinence (due to weak pelvic floor muscles) 14. Psychological disorders like depression 15. Psychosocial difficulties like social stigmatization « So how do you ask all these: start like this. + Ok Mrs. Kelly Lam going to ask few specitic questions about your present and past medical health. Just let me know if you have any problems, Ok (Oh! sure Doe) For how long have you really been concemed about your weight gain? ( May be from the past 6-7 months) What do you think is the major reason for your obesity? (I really don't know) + How is your appetite? (It's tov much Doe: ] want to stop eating junk fod but | eannot control} Ok for how Jong have you been having this increased appetite? ( For the last 2-3 yrs) Do you have any problems with your breathing: especially in the nights? (Nod How is your urination? (Pretty good). | mean did you notice any inereased frequeney {no}: have you ever leaked without your knowledge (n0) Do you have any problem with your bowel movements? (They are pretty rexular) [Note: You have to consider hypothyroidism and Cushing's syndrome as a differential diagnosis in a Doorway information Page 3 of 5 ease of obesity, "Have you ever had problems adjusting to temperatures"{ No}. "Have you been on any steroid medications for any reason?" (No). You already know that she did not have any problems with bowel movements (constipation in hypothyroidism)} + Did you notice any joint pain especially at the level of hips or knees? ( Some pain in both knees} + Have you ever been diagnosed with high blood pressure (no). When was your last visit with your primary care physician ( A couple of years ago } + Ilave you ever had any heart problems? (no) + Have you had any history of stroke? (no} + Have you ever been tested for diabetes? (no) + Ok Mrs, Kelly when did you check your cholesterol Jevel last time? (think S yrs ago and it was slightly elevated and after that I did some exercises, but it did not really help much) » Have you had any surg in the past? (Yes Doc. choleeystectomy 9 months back} + Are you alllergie to any thing? (no) Ok Mrs, Kelly now I would like to ask you few personal questions. Everything you say will be kept confidential. (Ok Doc sure) + How has been your menstrual cycles? (They have become irregular these day's. but they are not bothering me much.) + From how Jong have you been having these irregular periods? ( or the past 2-3 yrs, it seems everything started from the past 2-3 yrs} + When was your last menstrual period? (20 days ayo) # Are you sexually active? ( Thi one more problem for me doctor, these days | don't feel like having sex) + How is your mood Mrs. Kelly? Are you feeling Ok? (Tam feeling a little bit down these days) ‘+ Do you smoke Mrs. Kelly (No) + Do you drink any type of alcoholic beverages ( oevasionally 1-2 deers in the weekends) Ooorway infarmation Page 401 5 «Are you taking any prescri + Do you take any over the counter medical 4 Do you take any recreational type of drugs (10) « Can you deseribe me more about you diet? | mean what is your diet usually consists of? (Pretty much cheese and junk food doctor. some times fruits) + What do you do for your tiving? I mean do you work? (yes doc I am working as a desk elerk} Examination: «Just do some focused lung, ancl heart examin: «Check the thyroid gland. + Check extremities tor any edema. Counseling «+ ‘There is a possibility of thyroid problem (even Cushing's syndrome if the patient is on steroids) in your case even though very unlikely, first let me run some tests an you then we will sit together and go over the treatment options available. « Mean while try to restrict fatty food and start regular exercise. +» Most people will not succeed if they radicully change their current eating and cooking habits; however, you will probably have greater success if you try to modily only one aspect of your eating habits at a time, Eventually. you will find yourself eating a healthier diet cellent books available. + Ifyou would like more specific advice for diet changes there are many or you may wish (0 ask for a formal consult with a dietitian. Investigatis + CBC with Differentials ° EKG «Thyroid panel Doorway information Page Sof 5 + Urine cortisol levels «Lipid profile Diagnosis: + Obesity «Hypothyroidism # Cushing's syndrome Note: This i appropriate transition sentences and open eniled questions. a quick glance of questions that you have to ask in a case of obesity, Don't forget to use For a case of spells you have to consider these things specifically Page | of 4 Case of spells/loss of consciousness Syncope is the abrupt and transient loss of consciousness associated with absence of posturai tone, followed by a rapid and usually complete recovery, The common causes of syncope include Cardiac cause, most often @ bradyarrhythmia or tachyarrhythmia Neurally mediated cause - vasovagel syncope Neurological causes ike - TIA, stroke, seizures Psychiatric cause - Hyperventilation and conversion disorder Medications or toxins Unexplained syncope An accurate diagnosis of the underlying etiology for syncope can often be made from the history, physical examination, simple laboratory tests, and an ECG. The evaluation of the patient with syncope is the same as that for presyncope, which is the prodromal symptom of fainting. Such patients usually present with symptoms of dizziness Scenario: So basically ask these questions. + Can you please describe me the entire spell, starting from when you were feeling entirely normal? (Open ended question) « Ifhe doesn't cover what he was doing at the time of spell and how much time he lost his consciousness in his history, you need to take that history, Do you have any idea of what might be the cause of your spell? Is this the first of these spells; Have you had similar spells before? Was there anyone around when this occurred? What did they say about your spell? Are you back to normal/were you normal before the event and between the spells? Have you had any nausea or vornitings before the spell? Have you had any chest pain? Have you had any breathing problems? Have you ever had any palpitations? Have you noticed any weakness in legs and arms? Have you noticed any tingling & numbness anywhere? Did any one say that you had jerky type of rhytimic movements? Ask about any sudden visual changes or blurriness? Ask about any history of head trauma? Ask about bowel and bladder incontinence? Ask about the risk factors for stroke like diabetes, hypertension or heart problems? Ask all his medications including over the counter and illicit drugs. Ask was there any history of seizures In the past. ‘Ask about any history of anxiety or past psychiatric disorders. So these are the basic questions that you have to ask for any pt presented with spells (syncope). Don't forget to ask all the general information like family history, allergic history, and social history (smoking, alcoho!) just like for every patient Examination » Order orthostatic changes (both BP and HR) in the investigations section. For a case of spells you have to consider these things specifically Page 2 of 4 + Do complete neurological exam * Auscultate heart. © Check for carotid bruit. Investigations: « Electrocardiogram + 24 hr Holter monitoring or patient activated event recording Is commonly obtained in the assessment of syncope. « Echocardiography: An echocardiogram may diagnose underlying structural heart disease such as left ventricular dysfunction, hypertrophic cardiomyopathy, or significant aortic stenosis, + Neurologic testing: Neurologic tests, including electroencephalogram, brain CT scan, brain magnetic resonance imaging, and carctid Doppler ultrasound, are frequently obtained in patients with syncope. The gold standard for the diagnosis of seizures was a positive electroencephalogram without other apparent cause. + Exercise testing: An exercise test Is frequently obtained in patients with cardiac disease and also has a role in patients with a history of exertion-related syncope or exercise-induced arrhythmias. « Upright tilt table test: The tit table test hes become @ commonly performed test for the evaluation of syncope, particularly in young, otherwise healthy patients in whom the diagnosis of neurocardiogenic syncope is often entertained. « Ifyou are suspecting a drug abuse order Toxic screen. « Blood sugar and metabolic screen can be ordered if you are suspecting a hypogiyce electrolyte imbalance. + Gastrointestinal bleeding: A positive stool guaiac generally indicates gastrointestinal blood Joss, which can result sequentiaily in anemia, hypovolemia, and syncope or Explanation: ‘The clinical features associated with the syncopal event are extremely important. Number of episodes: Always ask about the number of episodes. Benign causes of syncope are usually associated with a single syncopal episode or with multiple episodes over many years. Associated symptoms: Symptoms occurring in association with syncope can point toward a specific cause. As examples: dyspnea may suggest an acute pulmonary ‘embolism; angina frequently indicates an underlying cardiac cause; @ history of focal neurologic abnormalities favors a neurologic origin; nausea, vomiting, Glaphoresis, and pallor after an episode suggest high vagal tone; and urination and defecation suggest, but do not prove, a seizure. Prodrome: Neurocardiagenic syncope (vasovagal syncope) Is usually associated with prodrome of dizziness, nausea, pallor, and/or diaphoresis. Such symptoms may also occur without an episode of syncope. "Auras" are associated with seizures. Position: Neurocardiogenic syncope commonly occurs when the patient is erect, not usually when supine. Syncope resulting from orthostatic hypotension is frequently associated with the change from a supine to erect posture. In comparison, syncope that occurs when the patient is supine suggests an arrhythmia. For a case of spells you have to consider these things specifically Page 3 ot 4 Warning: The sudden loss of consciousness without warning is most fikely to result from an arrhythmia (bradycardia or tachycardia) Preceding events: Coughing, eating, drinking cold liquid, urinating, and defecating can ali cause syncope; such etiologies are often referred to as situational syncope. Duration of symptoms: A prolonged loss of consciousness may indicate a seizure or aortic ‘stenosis. By comparison, arrhythmias and neurocardiogenic syncope are often associated with a brief period of syncope, since the supine position reestablishes some blood flow to the brain and can therefore result in the restoration of consciousness, This sequence of events can occur even if the arrhythmia is maintained Recovery: Persistence of nausea, pallor, and diaphoresis in addition to a proionged recovery from the episode suggest a vagal event. These findings are helpful in distinguishing neurocardiogenic syncope from syncope due to an arrhythmia Significant neurologic changes or confusion during the recovery period may be due to a stroke or seizure. Witness: A witness to the syncopal event may verify the loss of consciousness, any associated limb movements, and the presence or absence of pallor or diaphoresis. Information regarding the presence or absence of a pulse is also of great diagnostic significance. Exertional syncope: Among the pathologic causes of exertional syncope are ventricular tachycardia and obstruction resulting from aortic stenosis or hypertrophic cardiomyopathy, and hypotension due to vagally-mediated vasodepression in patients with hypertrophic cardiomyopathy. Age: Neurocardiogenic syncope is more likely to occur among young, otherwise healthy patients; however, syncope resulting from the long QT syndrome or hypertrophic cardiomyopathy can also occur in young individuals. Preexisting medical conditions: Coexisting illnesses can predispose a patient to experience syncope or can favor a specific cause. As examples: patients with psychiatric illness may have syncope secondary to hyperventilation or panic attacks; syncope in a patient with diabetes mellitus may result from orthostatic hypotension secondary to autonomic neuropathy; antihypertensive medications can result in syncope; and the presence of heart disease strongly implies @ cardiac cause for syncope Injury: Some investigators have suggested that patients with syncope in whom a significant injury occurs have a high mortality rate. However, syncope can also result from the high vagal tone that can occur with extreme pain from any cause. Medications or recreational drugs: Medications can cause syncope, especially antiarrhythmic (proarrhythmic effects) and antihypertensive agents (primarily due to orthostatic hypotension with sympathetic blockers). The abuse of illicit drugs or alcohol also has been associated with syncope Eating: A vagal “surge” immediately upon swallowing can cause bradycardia and hypotension in predisposed patients. Postprandial hypotension may also refect inadequate sympathetic compensation to meal-induced splanchnic pooling. Distinction of syncope from sei: One distinguishing feature is that patients ures For a case of spelis you have to consider these things specifically Page 4 of 4 with seizures rarely have an abrupt and complete recovery. Instead, the postictal state is characterized by a slow and complete recovery. Another important clue, if present, is evidence of soft tissue injury at multiple sites due to tonic-clonic movements during the seizure. PHYSICAL EXAMINATION: Abnormal vital signs: Blood pressure obtained in the supine, sitting, and erect position may detect orthostatic hypotension. turbances in heart rhythm or breathing: The heart rate may be rapid or slow gue to a number of possible rhythm disturbances, or irregular due to atrial brillation The pulse and blood pressure should be obtained when supine, setting, and erect. Cardiac auscultatory findings: The cardiac examination may reveal the murmur of zortic stenosis, pulmonic stenosis, or atrial myxoma. Pulmonary hypertension may be suggested by a loud, palpable P2. Abnormal neurologic findings: Unilateral abnormalities found upon neurologic examination may reflect a cerebral vascular accident, EEE A629 yr old male with terminal cancer requesting for pain medications Page | of 8 A 69 yr old male with terminal cancer requesting for pain medi If you get a case like this you really have to show empathy and care Start with formal greeting and place a hand on the paticnt shoulder, make an eye contaet then the ask an open ended question "Mr. XYZ please tell me what bothered you to come in here? (I am having pain in my epigastrium) Thave been informed that you have been diagnosed with cancer, Is that carreet? (yes) ‘Could you please tell me more about your cancer? (I have panereati months back) cancer. diagnosed 3 stand what Jam very sorry to here that (Thank you doe). | know its very difficult. | cam unde: you are going through. But [want you to know that | am there to help you if you need anything and make you feel comfortable, ( hanks you very much) Can you please explain me little bit more about your pain? How severe is the pain in a scale of | to 10? Do you think there is anything that makes your pain less? Do you have pain anywhere else? (Some times my hack hurts) ‘Are you using any medications for your pain especially any narcaties or morphine? ( not much) Do you have any other complaints other than pain? (1 am feeling tired most of the time) How is you appetite? ( Its very much decreased), Have you lost any weight? (yes: around 1-12 pounds in 3 months) Do you have any Fever? (n0) How are your bowel movements? Do you have any problem urinating? How is your mood? ( not good doe. [ feel depressed) Have you had any thoughts of ending your life? {not really so tar) Can you please tell me about your home situation? (| don't have anyone doc. 1 live alone) Do you have any one to help of support you like any friends or family members (| have few close friends, yes they certainly help if | need) 69 yr old male with terminal cancer requesting for pain medications Page 2 of 3 My. xyz will certainly help you in relieving your pain. | will preseribe some narcoties like merphine te relive your pain. I would also like you to be aware of certain things which will be necessary at some point in your life, 1 am very sorry 0 ask you these questions but | hope you understand the situation (Thank you doc, don't worry ask me) + Where do you want to live? Do you want to stay at your home or at aursing home? (I want to stay at home) + Are you aware about “hospice”? {not much) Ok let me expltin about hospice. Hospiee care is a choice you can make 1o enhance your quality of life in a terminal stage, You can also choose to die at home with the support of family, friends, and caring, professionals. Over 90% of hospice cate is provided at yout home. The advantage of Hospice care is that the providers have the skills and resources to permit you to live as pain-ftee, as comforzable. and as full a life as possible. In addit provide social, spiritual and physical support to you and your family. All hospice care is under prof con to providing pain relief Hospice care emphasizes cornfort measures and counseling: to ional medical supervision. So I strongly advise you 10 take Hospice care. (Thank you very much Doe, You relieved most of my tensions) © Okc are you aware of advance directives? (no not much doe), Ok, an “advance directive” ara “living will”, wil! enable you to give your opinion on hew you should be treated when you teach the terminal stage of the disease and not in a state 19 make a decision or you can give the right te take that decision to a loved one who you think will take a wise decision for you, Do you understand what I am saying? (yes doc) © Do you have any other questions? (No not much doc) Examination + Auscultate and palpate the abdomen, © Quick auscultation of the heart and lungs. + Look conjunctiva for pallor or jaundice If you do this much this is more than enough for the exam. «There is no single way to give counseling. his is an example for you to give counseling but bear in mind its not necessary that you follow this exactly word to word, This just gives you an idea to help you build up your own way in which YOU ARE COMFORTABLE, Its fine us long as you show that you are sensitive, supportive and conveying necessary information 69 yr old male with terminal cancer requesting for pain medications Page 3 of 3 + You don't need to write investigations or differential diagnosis if the problem is purely cancer A.62 yr old male with contusion Page | of 4 A 62 yr old male with confusion «Evaluation of confusion has very broad differential diagnosis. So narrowing down to differential diagnosis depends on the SP's complaints, + Start with formal greeting and as "What brought you in today?" (You know doe actually | don't think [ have any problem but my wife says Iam very confused these days) = Tow long has she been concerned about this? (J tbink from the past 2-3 months) «Is she saying that you are confused all the time or is there any specific time or related specific situation? (All the time Doc) »T understand that you are not much concerned about this. But let me ask few more questions to find out ‘what exaetly is going on. Is that ok with you?" (Sounds great oc) + Do you have any problems with your memory? Or has she ever complained about your memory? (No) * Do you feel any weakness in the extremitics? (No) « Doyou feel abnormal sensations like tingling or numbness in the extremities? (Nod + Do you feel dizzy? (No) + Have you ever had any fit like movements or seizures? (No} + Do you have any history of/head trauma? (No) + Doyouhave any fever? (No) + Do you have any headache? (No) + Have you ever passed out? (No} + Can yeu please describe me about your typical day. like your routine activities of daily living? (Note: Please know the Katz Activities of Daily Living (ADLs) - "DEATH" ie. Dressing. Ambulating, Toileting. Hygiene and Instrumental Activities of Daily Living (ADIs) - "SHAFT" Le, Shopping, Housekeeping, Accounting, Food preparation, and Transportation] ing, 4 62 ye old male with confusion Page ? ot 4 [Noie: Everyone will have allengie history. ph How is your bowel habit? They are pretty good) How is your bladder function? (Good) Did you notice any increased frequeney of urination” (Yes, I have this problem from long time: | usually pee little bit more) Do you have any history of diabetes? (Yes), When were you diagnosed with diabetes? (About 25 years ago). Are you on any medication? (Yes | am on insulin), Do you know how much insulin you take daify” (Usually my wife or my daughter gives check your blood sugar? (Rarely like once or twive in a year). 1s your blood sugar under control? sulin shots}, How ofien do you (Some times), Have you ever been admitted in the hospital for diabetic related complications? (No) or for any other reason? (No, Never) Do you have any other medical problems? (Yes f do have high blood pressure}, For how long? (Same as my diabetes), Are you taking any medications for that? (Yes; Lam on atenolol 25 mg twice daily; | guess), For how long have you been on these medications? (Around 18 years), How often do you check your blood pressure? (Once or twice in a year). Is your high blood pressure under control? (Not always, only seme times) Have you ever had any heart problems? (No) Have you ever had stroke? (No) Other than insulin and atenolol are you taking any other medications? (No) OK, Are you allergic to anything? (Yes: te penicillins) se ask about the allergic history. you don't have to get into details about the allergy if the case is not related to that, as in this case.] Ask about the family history? Are you sexually active? (No) Ta never) fou ever heen diagnosed with sexually transmitted diseases especially like syphilis? (No. Do you smoke? (No) Do you drink any type of alcoholic beverages? (no) 4A 62 yr old male with confusion Page 3 of 4 + Do you take any recreational drugs? (no) + Can you pleuse deseribe about your dietary habits? ( T usually eat beef and some fruits. I's just regular diet Doc} Lt you ask this much in the history that is more thaw enowgh. You may not get enough positive history for any particular diagnosis. So don’t worry Examination: + You have to do Mini mental MMSE. atus exam (MMSE) - Tais kind of people usually have normal «You must do fundoseopy + Check gait. musele strength, reflexes. sensations, and very quick eranial nerve examination + Iyou have time auscuhtate heart and lungs. Most of the times people wont get suflicient time because of the MMSE and CNS exam, Do us much as you can very fast. Once again practice repeatedly otherwise its very difficult to manage in the exam. Investigations: + CBC with differential + Serum electrolytes or Basic metabolic profile (BMP) + EKG and 24 hr Holter monitoring if there is any history of spells + Carotid doppler = CT head Urine toxicoiogy sereen if you suspeet any drug abuse Differential diagn + Insulin induced hypoglycemia + TIA A 62 yr old male with confusion Page 4 of 4 Muli infaret dementia :Iectrolyte abnormal Substance abuse or Drug intoxication Alzheimer's dementia We don't think you will have enough time to give counseling. Bul any way always ask "De you have any questions?” and formal "closing the encounter", Explain about the importance of tight blood sugar and hypertension contre “Lam sorry to here that your blood sugar and blood pressure are not under good control. Controlling blood sugar requires determination. J will give you a treatment plan, which advises on right diet. sels are exercise, and using the medication, High blood pressure could indicate that your blood v ‘having trouble. Hypertensiow/high blood pressure could complicate a diabetic’s problem, It coulei cause stroke, affect the fanetioning of the heart and even the kidneys, Again ¢ lar exereise, reduction of ight (if the patient appears over weight), limiting salt in your food could help in keeping hypertension in check. | would strongly advise you to take regular health maintenance examinations and tightly contral your blood sugar and blood pressure. What do you say Mr. xyz?" (Sound: A 25 yr old male for pre-employment check up Page | of 2 A 25yr old male for pre-employment check up You may get this kind of cases as either “Pre-employment” checkups or as an "Insurance checkup". They will tell you everything what you have to do some may’ be asked to fill up a FORM (It will be provided to you). In case you were not provided with a form you will have take a simple general history. Most of the times you will be asked to do physical examination only. If they want you (o do any specific examination. do that anc! do all the things that are mentioned inthe FORM first. You can fill the form after coming outside, In ease if the SP asks about the nish “Iilling"( examining ) the form form tell him that you will post it to his home. After you ask “Do you have any questions?" If he asks for some thing then ask the refevant history. If they doa’t tell anything specifically you can start taking history in brief. Ask cardinal symptoms of cach system Do you have any cough? (No). Do you have any problems with breathing (Ne} + Do you have any chest pain? (NO) + Do you have any headache? (No) Do you have weakness in th extremities? (Nop + How is your bowel habit? (Pretty good) + How is your bladder function? (Good) Then you must ask PAM HUGS FOS ieohol and sexwal history + Please do not forget to ask about Allergic, smoking, Here is the sample of the form that you might get. Il may not be exactly like this its just to give you an idea, You may get some of the components of this form Ifthey ask you to measure blood pressure (on the form/doorway inlormation) you have 10 measure, This is different from ail other cases where you need not to check blood pressure, After finishing the case you just have to fill out this form, You don't need to write any history (IF they don't ask) or things that they have not asked you. Ail you have to do fill out the form. 25 yr ofd male for pre~employment check up ‘Height Weight ‘Blood pressure Pulse rate Lung auscultation Heart auscultation i I t ispine | amin Page 2 of 2 Case of Abuse? Page 1012 Cave of Abuse: + There are many ways to ask direct questions that elicit a response about domestic viole abusive relationship. No single question is right as long a ce oF ‘ou are sensitive and supportive + Inceal life many battered women may hesitate to initiate information about abuse, but are relieved to answer when some one asks. That is why they (CSA exam) expeet you to know this kind of cases and usually you may get a case like this. So know everything how to ash and elicit a proper history * Ifyou are suspecting a domestic violence or a case af abuse ( Lx, Women with bruising or some pts with depression) you can start with a good sercening qt like this. + "Idon't know if this is a problem for you, but becaus so many people we see are clealing with abusive relationships, | have started ashing about it routinely. Are you currently in a relationship where you are physically hurt, threatened or feel afrsic’?* # Ifthe chief complaint itself is an abuse y 1 can ask direct questions like © "What happened’ How were you hurt? © "Were alcohol or drugs involved? How ? By whom? or Does your partner use drugs or abuse alcohoi? © “Has a weapon ever been used in an attack on you' © "How tong have you been in this abusive relationship? Has it happened ever before? Are you affaid it will happen again’? © "Has your partner ever made you have sex when you didn’t want to?" © "You have mentioned your partner looses his temper with you, How are things between hien and your children?" © "Have you ever lefl home? When? if not, have you wished to leave? What has prevented ae 9 “Are you planning to leaveldivor on your partner © “Has your partner ever threatened or tried (0 commit suicide © "Do you. yourself, think of suicide asa way out of the relationship?" If the answer is'y ask "Do you have a plan or method by which you would kill yoursel!? or partner?" > "Do you have an emergeney plan if needed’”* © “Are your family or friends aware of your situation?" + During the whole encounter tell her repeatedly that she does not deserve to be beaten, Battering is against the law ‘+ Examination is basically examining the injured (painted parts!) parts and quick auscultation of heart, lun the chest und having breathing problem) sion unless pt has a bi (no pore and abdomen (do palpation also) Counseling: nuise Ov Case of Abuse! Page 2 of 2 Note: ‘Counseling is a major part if you get a case in the exam Create a safe space by assuring her that you will do everything possible 1o maintain her safety ‘Assure her that her medical condition will be treated appropriately, and that she will not be forced todo anything against her will Teli her that children are cared for and safe (if present). Assure her confidentiality. Explain her that only with her signed consent will her medical records be released to any other source. fell her that violence never ends on its own and that the violence almost always escalates in severity and frequency over time, and that the only way to end itis 10 get away from the batwerer Always be respectful and non-judgmental; Say "I believe you, it is not your fault you are not crazy, you are not alone. help is available for you." Before sheyou leaves ask "Do you think is it safe to go home? Do you have u safe place to stay’? Would you like to speuk with a domestic violence counselor?" If you notice multiple bruises (nice painting by CSA people) and the patient did not give any history of abuse, ask like this "I notice you have a number of bruises. Did some one do this to you? It looks like some one hurt you. Can you please tell me what happened to you?" Patient with Insomnia: Page 1 of 4 Patient with Insomnia: Insomnia has numerous, often-coneurrent etiologies. including medical conditions, medicatio psychiatric disorders and poor sleep hygiene. Sleep apnea should be also considered in the differential diagnosis. problems in the USA and its good! to know about this ease for CSA. $o the evaluation usually requires detailed history taking. This is one of the most common Starting encounter with the SP: © What brought you in today? (T have problem with sleep Dec) ‘an you please tell me more about your problem? (use to Work as ituck driver during night and now I switched 1o day time work. since then Tam having problems with sieep. I think this all is due to the shift of work, Please give me some sleeping pills Doc) = Tunderstands that your problem might be due to change of working schedule bus there are some other common things and conditions that can cause sleep problems, which can be easily treatable if we can find. So I necd to ask few more questions about your siecp patterns, your general condition and some other things. Is that ok with you? (Sure go ahead Doc) + How long have you been having problem with sleep? (Around 3 - 4 weeks) + Do you huve problems with falling asleep? (Yah most of the times) + Do you have any problems with staying asleep? (Yah some times} + Do youhave problems with waking from sleep? (No) + You said that most of the times you are having problems with falling asleep. When do you asually go to bed? {between 8 to 9 PM), How much time do you take to fall into sleep?(1 (@ 2 hours). What do you do before you go to bed? T mean some people do exercise in late evening and drink alcohol before go to bed. Do you do any exercise like that? (No). Do you take any aleohol before you go to bed? (Yah). low much do you take? (A couple of beers usually). Do you smoke befare you go to bed? (Yah mostly after having dinner}, De you take caffeine or excess coflee before go to bed? (Not really) + Do you wateh television while lying on the bed?) (Vuh usually) «You said you also have some problems staying asleep. Do you wake up several times during night? (Not several but 2-3 times and if 1 wake up is takes awhile to go to sleep again). Ok you Patient with Insomnia’ Page 2 of 4 said you wake up 2-3 times in « night, do you have any idea what might be causing it? 1 mean do you wake up to urinate many times or any problems with breathing or coughing’ (No) [If you are asking several question always give a pause to each question] + Tlave you or any of your family members noticed frequent movement of legs during your sleep? (No. not that | know of) + Do you have pain anywhere’ (No) * How is your mood? (Pretty good) [If the SP appears depressed you have to ask all depression questions, But itis very unlikely to get a case with 2-3 problems like depression, shifting of work ete, as its very difficult to manage in 15 minutes] + How are your bowel habits? (Pretty good) ‘+ How is your bladder function? (Pretty good) + Do you have any other medical problems? (No.1 am pretty healthy) + Have you ever been hospitalized for any reason? (No, never) + Do you have any allergies? (No) + Are you taking any prescription medieations? {No}. Any over-the-counter medications? (No). Are you using any reereation type drugs” (Noy + Are you sexually active? (Yes). Do you have any problems with sexual performance? (Noy + You said you have a habit of smoking and drinking aleohot? How long have you bees smoking (25 yrs). How many cigarettes de you smoke in a day? (StoL0). How long have you been drinking alcohol? (Same like smoking), How much do you drink per day? (AS I said 2-3 beers a day) + Do you have any stress at your work? (No) + Ask about family history. [Note: Some times the major cause of sleep disturbance in middle-aged women is the menopause-related hot flush." Revent studies indivate that nearly every hot flush promotes ar arousal lrom sleep. So please kcep this in mind if'you get a female patient of menopausal age.] Patient with Insomnia: Page 3 of 4 Examination’ Cheek thyroid ‘Auscultate lungs and heart quickly Diagnosis: Circadian rhythm sivep disorder ‘Counseling: "Based on your history L think your problem is most probably due to a combination of multiple actors. Obviously shilt of work is a consideration bat at the same time maintaining sleep hygiene is also very important and it will certainly help you in recovering lrom your problem. | would like you to maintain a sleep diary tor two weeks to see the sleep pattems, Please keep regular fedtimes and wake times, even on weekends and days off from work, Limit or stop the use of nicotine, caffeine and alcohol, Exercise regularly, but no later than late aftemoon or early evening. Do not use the bed as a place to worry especially about not sleeping. Hf you feel negessaty, write down all your worties and concems before you go to bed and place the list on your dresser to examine the next morning. Use the bedroom only for sleep. Don't read. wateh television, cat or do other acti in the early aftemoon and for no longer than 30 minutes per day. Hat a light smack (ut not a ties in bed. Try to avoid daytime naps. But if you mast nap. do so heavy meal) before bedtime if food is needed because of hunger. Get regular exposure to outdoor sunlight especially in the late attenoon.” If you follow all these you may no longer have problem with sleep. We usualy don't recommend medications us this problem offen resolves with these behavioral modifications, Ok Do you have any questions? Explanation about insomnia: Certain medical conditioas. such as COPD, GERD, peptic ulcer disease, BPH (resulting in overflow incontinence) and congestive heart failure with associated paroxysmal nocturmat dyspnea, frequently disturb sleep and may be interpreted by the patient as insomnia Patients with chronic pain, such es that resulting from chronic pain syndromes, fibromyalgia, caneer pain may have insormnia and early-morning awakening. They (SP) wont tell unless you ask specifically about the pain, Patient with Insomnia’ Page 4 of 4 A psychiatric disorder, such as depression. is frequently a cause of chronie insomnia, especially in the elderly. Periodic fez movements during sleep are common in persons over 65 years of age. Although these limb movements are often associated with brief arousals, many patients have no sleep symptoms. Regardless of the eause of insomnia, most patients benefit from behavioral approaches that focus on good sleep habits. Exposure to bright light at appropriate times can help realign the circadian rhythm in patients whose sleep-wake cycle has shitled to undesirable times Chronic insomnia may reflect a disturbance in the normal cireadian sleep-wake chythm as in this patient A.69 yr old tady with Dizziness: Page 1 of 5 A65 yr old lady with Dizziness: Not only in CSA exam, even in real life dizziness is both challenging and ftustrating for the clinician. Dizziness is a nonspecific term. Vertigo, non-specific "dizziness", disequilibrium, presyncope or near syncope can all be complained as a simple word "Dizziness" by the patients, Obtaining an extensive history és most important to put the patient into one of these more specific categories, Encounter with the SP: + "What brought you in today?" (1 feel dizzy) © "Can you please explain me litte bit more about your dizviness?" (I always feel dizzy. I don't know what you want me to explain) 4 Makes good eye contact and say, "Well... Ht looks like you are not in a good mood. Tam here to help you. Are you comfortable madam Is there anyway that | ean help you?" (My mood is ok just help me from this dizziness) + "Ok, To understand that most people with dizziness are not happy. But | want you to belp yeu. ‘To clearly understand your problem [ need to ask few questions very quickly is that Ok with you?” Note that she wont be happy all the time during this encounter. She could be an. uncooperative Patient + "Tell me what do you mean by dizzy?" (Dizzy means dizzy) + “Well, Many people describe their problems as dizay though its not real dizziness, Anyway’ when you get dizzy do you feel like room is spinning around you?" (No) or "do you think that you are spinning inside?" (Yah) + "Is your dizziness constant at all the time or does it just eomes and goes" (Comes und goes) + "When did dizziness first occur?" (2 weeks ayo), "How offen do you get (Once or twice in a day), "How long do they last?" {1 to 5 minutes) + "Do you have any warning signs that the attack is about to start?" (No not eully). "Do they occur at any particular time of the day or night?" (Lam aot sure) + "Does change of motion make you dizzy" (1 don’t know) + "Do you know of any possible cause of your dizziness?" (I don't know) A665 yr old lady with Dizziness: Page 2 of 5 + "Do you know anything that will stop your dizziness or make it better? + "Do you know anything that will make your dizziness worse?” (No) + "When you get dizzy do you have a tendeney to fall?" (Yes} "To which side? Is it to right? Or Left?” (1 don's remember} + "Haye you ever lost consciousness?" (No) + "Do you have loss of balance when walking?" (Yah sometimes) + “Have you had any headache when you get dizzy? (yah light headedness) "Do you have it now?" (No), "Have you had any vomitings when you get dizzy?" (No), "Have you had any palpitations or shortness of breath or « feeling of panic when you get dizzy’! (No) + "Have you noticed any difficulty in hearing? (No). + "Do you hear any ringing in the ears?" (No) + “Do you have any problems like double vision or blurry vision” {No} + “Have you ever noticed any weakness in the arms or legs?" (No) + "Do you have any numbness on face or arms or legs?" (No) + "Do you have any other medical problems other than dizziness?" (1 have low blood pressure). Have you had any heart problems?" (No) + "Have you ever been hospitalized?" (Yes for stroke few years ayo} + "Have you ever had history of trauma to your head or neck?" (No) + "Are you allergic to anything?" (No) + “Are you taking any prescription medications now?" (Yes aspirin), "Any over the counter medications?” (No) + “Do you smoke?" (no), "Do you take aleohol?" (Yah social drinking, | to 2 beers in the weekend) + "Do you have any problems with your bowel movements?" (No} A865 yr old lady with Dizziness Page 3 of 5 = "How is your bladder fin ction?" (Good) + “How has your appetite been lately?" ( od) + Ask about family health, Examinat = Do complete quick CNS exam, which must include nystagmus. gait, Romberg’s test, and cerebellar function tests. Note: Not only in the CSA exam people with dizziness are often reluctant to move because of the fear of fall, though they are able to walk, Before you test gait or Romberg's test you have to say " can imagine how uncomfortable it assist you, | will help you in every aspect of the examination, This wont take more than & couple of minutes." but Jam here to + Auscultate for "carotid bruit" © Auseultate heart + Hearing tests - If you get a positive history of hearing loss + Do etoscapie examination it out got any positive history like ear discharge. pain in the ears. ringing in the ears and aural fullness «As Tong as memory is intact you don't need to do MMSE. and obviously you don't get time, So always do most important things first Differential diagnosis in general for dizziness case: + Benign positional vertigo + TIA + Stroke + Postural hypotension/Orthostatic hypotension Archythmias A 65 yr old lady with Dizziness: Page 4 of § + CNS tumors Meniere's dis «Drug induced/Polypharmacy: + Nonspecific dizziness + Peripheral neuropathy + Thyroid abnormalities © Anemia © Metabolic di srbances like hypoglycemia Investigations: + Check orthostatic hypotension “BE with differential «Basie metabolic profile (Na, K. Cl, CO2. BUN, Cr, Calcium and blood sugar). + Thyroid function tests + Carotid Doppler If you get a relevant history for stroke/TLA + MRI of brain - For suspected acoustic neuroma or any CNS tumor © 24hr Holter monito In patients with H/O palpitations and cardia Note: If you get an uncooperative patient it is very difficult to get everything in 15 min as they don't ‘cooperate and don't do all the tests properly. So don't get pantie, try to do as muich as you ean and remember that they are told to aet like that. This problom is for all the CSA takers. So again the key to pas this exam is practice, practice and practice. Short explanation about dizziness: Ve indicating excessive autonomic activi objective sign of vertigo is nystagmus. examination of eyes is extremely important and they expect you $ a mildest form, itis usually associated with nausea, vomiting. and perspiration, y. Typically there is no loss of consciousness. Be 10; Unless i se the only to check the eyes, You have to tell the patient that you are checking the eyes. If you just examine 65 yr old lady with Dizziness Page 501 5 without informing you don't get point for that. Presyncope: Its a perception that he or she is about to faint, Pati nts may have associated symptoms like constriction of the visual fields, diaphoresis, and nausea but there is no actual Joss of consciousness Disequilibrium: ‘These patients typically do not have problems when sitting or lying down but notice unsteadiness when standing or especially during walking, ‘These patients also do not lease consciousness. Heel pain of Foot pain Page 1 of 5 Heel pain (Or) Foot pain: + Ask about the location of pain + Ask about the intensity of pain «Ask about the quality of pain + Ask about the origin and ducation of pain # Ask about the radiation of pain «Ask about the aggravating factors in detail ( walking, standing, “After standing, how long does it take for tae pain to start?” ete.) # Ask about the alleviating factors + Ask about the prior episodes and frequency # Ask about any trauma to foot + Ask about the associated factors like fever. and other joint pains ¢ Ask about the morning stiffness ‘+ Ask about the prior 1VO diarrhea or any acute illness (For possible reactive arthritis) + Ask about the urethral discharge and conjunctivitis ( For possible Reiter's syndrome) + Ask about the H/O rash (For psoriatic arthritis) + Ask about the occupation and type of work in detail ("What type of werk do you do? Does your work involve any prolonged standing? Do you have to walk a lot at you work?) ‘+ Ask about the family 1/0 rheumatoid arthritis or any other joint diseases # Ask about any other medicat problems = Ask about medications # Ask about allergies Heel pain of Foot pain Page 2 of § + Ask about sexual history # Ask about social history Examination: + Inspection of the foot (Let the SP know that you are inspecting). + Palpation of the entire tvot (not just the heel) for any point tenderness ‘+ Check for the range of motion al the ankle & forefvot joints. Note the pain & restriction of movements, + Ask the patient to do active dorsiflexion and plantar flexion and see for any tendon tendemess (Bor tendinitis), Check the eyes for possible conjunctivitis (If yau suspect Reiter's syndrome) Investigatis © CBC with diff + X-ray of foot and ankle 3 views: # Rheumatoid factor assay Differential diagnos + Plantar fasciitis + Caleaneai periostitis # Caleaneal spurs Painful heel pad syndrome Bone tumors Rheumatoid asthritis, Reiter's syndrome Heel pain of Foot pain Page 3 of 5 Counseling: + Afier the appropriate closing you can counsel like this "I have to order X ray of foot, ante and some basic blood tests before we come to 2 proper diagnosis. Mean while I will try to help you to relive your pain, Give 2-3 days of rest to your foot. apply iging for 30 minutes for every ¢hours and use soit heel pads, At the same time take over the counter ibuprofen 400 mg 3 times a day ‘You can also try resting padded foot splint; these splints are available in pharmacies that feature orthopedic si plies. Wearing slippers or going barefoot may result in a recurrence of your symploms, So your first step out of bed should be made with a supportive shoe of sandal on. You! have to avoid excessive heel impact from jumping, walking. Most o ‘our tests shows abnormal results we the times people will get better with these measures. If you don't getter better or will sit together and will discuss the other possible options," Is that ok with you? (Sounds great Doe) «+ Good thank you, Do you have any other questions? Short explanation: is, and bursitis «Disorders of the ankle and foot, including ankle sprains. tendinitis. plantar fs usually occur in association with movement and result from trauma. «+ The common underlying problems include osteoporosis, spondyloarthritis and enthesitis Ginflammation at muscle insertions) of lower limb structures, improper footwear, moving from a home with wood flooring to one with a slab foundation. prolonged standing on concrete floors, and structural disorders such as joint laxity or malalignment of the lower limbs, Plantar fasei + Plantar fasciitis is one of the most common causes of foot pain in adults, Heel spurs often coexist and may represent a secondary response 10 an inflammatory reaction. The predominant symptom of plantar fasciitis is pain in the plantar region of the foot that is worse when initiating walking. A hallmark for diagnosis of plantar Fasciitis is local point tenderness. ‘This is best elicited by the examiner dorsiflexing the patients toes with one hand in order to pull the plantar faseia tavt, and then palpating with the thumb or index finger of the other hand along the fascia from the heel 10 the forefoot. « Inaddition to soft tissue pain or fasciitis, plantar heel pain may be induced by a number of other disorders, These include nerve pain due 1o enttapment, hone pain tendinitis of the posterior tibia ndrome and jon. Reiter's s or flexor digitorum longus tendons, or the presence of tumor or inf Heel pain of Foot pain Page 401 5 other spondyloarthropathies, Loigren’s syndrome, and sareoidosis should also be considered in the differential diagnosis. «Radiographic s tudies are usually recommended in patients with persistent plantar pain after a tral of conservative therapy. Lateral and axial radiographs are the appropriate views due to the myriad of diagnosti possibilities. The primary goal of radiography is to rule out other disorders, + The sof tissue may be evaluated with MR imaging in cases that are resisiant to treatment ‘Technetium seintigraphy has also been succes fl in localiving the inflammatory focus and raling, ‘out stress fracture, Cateaneal periostitis « Calcaneal periostitis may result from trauma, Reiter's syndrome, ankylosing spondylitis. psorin arthritis, or rheumatoid arthritis. Pain is usually bilateral. along the lateral and plantar aspect of the hoes, worse in the morning upon arising, and often accompanied by moming stiiTness Tenderness is diffusely prevent along the plantar aspect of the heel and midfoot, and along the lateral horder of une heels. «Bilateral involvement should alert to the possibility of a coexistent systemic rheumatic disease. Caleaneal spurs # Calcaneal spurs can develop on the plantar tuberosity and extend across the entire width of the calancus. ‘« Heel spurs are typically asymptomatic; pain occurs when the apex is angled downward by depression of the Jong arch, An acutely painful heel spur may also be seen in certain systemic Reiter's syndrome. or theumatoid arthriti diseases. such as ankylosing spondylitis Painful heel pad syndrome «+ ‘The paint hee! ped syndrome most often occurs in marathon runners. [Lis thought to result from disruption of the fibrous septac that compartmentalize the fat in the heel pad. Pain is localized to the heel pad; the plantar fascia is not tender and pain is not accentuated ais the examiner dorsiflenes the toes, Henne TU EEE ‘ource: Reproduced with permission from UpToDate If you get a case of low back pain consider these things Page | of 6 Low back pain: Ifyou get a case of low ack pain cons ler these thing: + Ankylosing spondylitis + Herniated lumbar dise Spinal stenosis Osteoporosis with vertebral crush fractures Metastatic bone cancers Primary bone tumors like multiple myeloma Rheumatoid arthritis Examination’ + Check spinal tendemess + Check paraspinal tenderness Check lower extremity pulses # Check sensations of the both lower extremities Do complete motor and reflex testing of the both lower extremities Do straight ieg raising test «© Check lumbosacral spine range of motion Check the gait Investigations: «CBC with diff. + HLA-B27 It you get a case of low back pain consider these things Page 2? of 6 ‘+ Rheumatoid factor levels + X-ray of the lumbosacral spine + MRI of the spine Upper extremity p: [f you get a case of upper extremity pain consider these issues: ‘+ Carpal tunne! syndrome (ask about the occupation) ‘* Cervical spondylitis Herniated cervical dise + Thoraeie outlet syndrome (ask whether the symptoms worsen with the ahove head activities like combing) tis + Tenosyno Trauma 4 Referred pain from coronary ischeraia In the history just follow LIQOR AAA and PAM HUGS FOSS Examination: © Cheek the thyroid gland. © Check the neck movements and the range of motion + Do thoracie outlet west ( Adson’s test) - Ask him to take the deep breath, extend the neck and tra the chin towards the opposite side and repeat the st with the chin opposite side. In the presence of thoracic outlet syndrome the radial pulse will disappear + Do Phalen’s test (lor carpal tunnel syndrome) - Hold the patients wrists in acute flexion for 30-60 seconds, Patient will complain of pain, aumbness and tingling over the distribution of medial nerve if the test is positive. If you get a case of low back pain consider these things Page 3 of 6 « You can also elicit Tinel’s sign also if you want. With your finger percuss over the course of medial nerve in the carpal tunnel, Patient will compiain of pain, numbness anc! tingling over the distribution of medial nerve if the testis positive + Check sensations. do motor and check reflexes of the upper extremities both sides. Investigations: = CBC with differential, ESR EMG and nerve conduetion studies + X-ray of the cervical and thoracie spine © EKG MRI of the spine A125 yr old female with nausea and vomiting «First think about the common causes of nausea and vomiting in this age group + Gastroenteritis (food poisoning) - Ask about having food out side / travel history’ any other members being ill/ associated symptoms like abdominal cramps and diarchea «+ Obstructing disorders - pyloric obstruction (classically vomiting withia 1 hy after having food). intestinal obstruction (vomiting late post prandial perind}’ constipation! is she passing gas (don't ask flatus: if you ask flatus SP will say 'what is that?'*}. Relief of the pain with emesis is very characteristic of smail bowel obstruction. Vomiting kas no effeet on acute pancreatitis or cholecystitis. «+ Inflammatory diseases (cholecystitis - pain in the right bypochondriae region. acute panereatitis - severe epigastric pain radiating to back, appendicitis - initially periumbilical pain later to right lower quadrant pain) + Impaired motor fnetion - Diabetic pastro paresis. GERD ( So ask about any history of diabetes may be type | in this female} « Intraccbral disorders like malignaney, infections - So ask about fever, headache and the quality of vomiting (projectile or not) if you get a case of low back pain consider these things Page 4 of 6 Please follow the history taking Exant Investigations + Drugs (diogxin. cancer chemotherapy) - Just ask are you taking any medications. « The 2 most common conditions and you will be likely to be tested in the CSA are pregnancy and anorexia nervosa. You should not forget fo ask about the LMP because if you get a ease of nausea and vomiting in the CSA it is most likely a pregnancy and in fact the SP may ask you "Doc is ita pregnaney?" «Always follow the PAM HUGS because post operative vomiting. ethanol intake. drug: diabetes. ORGYN history (LMP) will cover automatically and you don't need to remember what to ask + Don't worry about the rare causes (labymthitis, uremia, adrenal insufficiency) of vomiting for this exam. tion of the site for complete description of vomiting # Auscultation of the abdomen (decreased bowel sounds indicates ileus, increased bowel sounds. indicates bowel obstruction) « Abdominal palpation both superlicial and deep Fundoscopy if you ate suspecting intracranial causes « Explain about the need of rectal examination and pelvie examination (pregnant Quick Lungs and heart exam « Finally if you have some time look at the oropharynx «© CBC with ditt « Pregnancy test (rust for this female) « FOBT + Serum amylase and lipase (if you are suspecting) acute pancreatitis If you get @ case of low back pain consider these things Page 5 of 6 «+ Ultrasonogram (acute cholecystitis) + Blood sugar for DM + EKG should be obtained if patient has risk factors for MI (some times inferior wall MI presents only with vomitings especially in diabetics) but do not order for this female unless patient has ntractable vomitings and you are worried about hypokalemia A34 Yr O/F came for Bronchial asthma drug refill «© This case is not that mach important but always there is a possibility, so just have a look «The things that you need to ask specifically for this ease are «Start with formal greeting. ask open ended question like "What brought you in today?" + °Can you please tell me more about your asthma?" Like "When were you diagnosed fir the First time?" and "How have you been doing since then?" + "Can you please tell me about your current medications or” What medications are you on?" « "Did you notice any problems or side effects with your medications” + "Do you have any trouble breathing during the day or night with regular activity 2" (yes) + "How often does this occur on a weekly basis?” * "Do you have any trouble breathing with exercise?” (yes) + "How often does this occur ona weekly basis?” + "Do you have episodes of excessive coughing during the day or night time?" (yes) "How ofien does this occur on a weekly basis” © "Have you ever been admitted in the hospital for an acute or severe attack?" + "Tell me what do you think abou the severity of your asthma? Do you think is it getting hetter or worse? Do you know what precipitates your asthma?" # "Are you taking any precautions to avoid those?” If you get a case af low back pain consider these things Page 6 of 6 Afier this you will just have to follow PAM HUGS FOSS. Make sure you ask smoking history and tell about the importance of smoking cessation Examination basically requites HEENT to look for any sinus tendemess (sinusitis). signs of upper respiratory tract infection, which can aggravate or precipitate asthma, You have to do complete Sung. examination. Look for JVD and pedal edema for signs of cor pulmonale even though it's a very rave complication of asthma. Spirometry or pulmonary fanetion tests are usually not required unless the patient is elderly and having persistent asthma. CBC with differential. Aspergillus serology, Chest X ray. X-ray of Para nasal sinuses, 24 hr pH for GERD and skin tests are required in chronic persistent and refractory asthma. Differential diagnosis for chronic persistent asthma in a smoker includes 1. Bronchial asthma corp 3. Broncho pulmonary aspergillosis 4, Sinusitis, Atypical GERD Pann 1 oft 30 Yr. O/F Complaining of Abdominal P. Vitals «= Pulse--98/min © B.8--120/75 mm of Hg » Temp-101.3, # Rrate--22/min Make a mental checklist of Differential Diagnosis * Pelvic inflammatory disease « Pelvic abscess «Endometriosis « Urinary tract infection + Appendicitis + Rupture/torsion of ovarian cyst * Acute cholecystitis + Renal colic * Ectopic pregnancy = Abortion + Acute gastroenteritis «Inflammatory bowel disease 8 Page | of 1 S.P's Notes If the doctor asks you anything other than these just say ‘no’ (or) say things that are normal in daily routine life. + You are Mrs. Mary, age: 30yrs « Have abdominal pain since 12 brs « Started slowly, progressively increasing + 7-8/10 in severity «Right below the umbilicus «It’s a type of sharp pain * All over your lower abdomen + Began after eating a large meal ¢ Moving around makes it worse + No alleviating factors «Not associated with vomiting but have nauseating feeling ‘« Passing urine more number of times and have burning urination + No bowel problems + Last menstrual period was 3 weeks ago + No discharge from vagina/no bieeding from vagina «Have fever since yesterday associated with chills and rigors ‘+ Have one episode of urinary tract infections (UTI) in the past « No allergies Once hospitatized for evaluation of UTI + Have multiple sexual partners + Using oral contraceptive pitls # Families’ health is normal «Smoking - Ne + Alcohol- No . » Recreational drugs- No «Occupation: Working as a receptionist '» Appetite and wt is normal «© No illicit drug intake Ask this qt - Doc is it an appendicitis? S.P 's Check List History Taking Asked about : pain ( location, intensity, quality. the origin and duration, the progression, any radiation } aggravating and relieving factors; any voruiting, fever. urinary and bowel problems LMP: vaginal dischas Past History ‘Asked about : similar episodes in the past; history of allergies; past medical problems ( high BP, diabetes, kidney problems, urinary tract infection ) previous hospitalization ( surgery). family health appetite and change in weight, smoking, alcohol, ob/gyn history sexual history, medications, occupation Examination Washed hands- Auscultated abdomen- Palpated abdomen superticially und deeply Check rebound tenderness- Looked for CVA tendemess- Performed Psoas sign and obturator sign- examined without gown, Counseling Explained physical findings and probable diagnosis- explained further work up ( blood test. urinalysis. ultrasound. abdomen x-ray) D.D for this case Pelvic inflammatory disease Pelvic abscess Urinary traet infeetion Appendicitis Rupture/torsion of ovarian cyst Investigation rectal and vaginal exam- CBC with differential coum- Urinalysis~ Pregnancy test- Abdomen x ray- Ultrasound abdomen vaginal bleeding ° Page 1 of 1 Sample patient note C.C: A 30 ¥/0 WF with abdominal pain. HPI: A 30 ¥/O WF who has a H/O UTI, pyelonephrtis who is in her usual state of health until yesterday started to have abdominal pain right below the umbilicus. The pain started after having a heavy meal; She describes the pain as sharp, 6-7/10 in severity, gradual in onset and progressively increasing, Later on, the pain moved to the lower abdomen. Moving around makes the pain worse; denies any alleviating factors. The pain is associated with nausea and 2 episodes of non-bloedy vornitings. She is also C/O having frequent burning urination, which started at more or fess same time, She also has fever associated with chills and rigors. ROS: She has regular bowel movements; no diarthea/constipation. She denies resent change in appetite and weight. Rest Is unremarkable. UTI one episode, Hospitalized once for evaluation of possible pyelonephritis. NKA Working as a receptionist. She never smoked ner had alcohol, SxH: Multiple sexual partners, her partner doesn’t use condoms, uses oral contraceptive pills. Never been tested for STDs. FH Both partners are laive and healthy. Ob & Gyn: LMP 3 weeks ago. No priors STD's, No H/o vaginal discharge PE: Vitals: Pulse 98/min, 8. -120/75 mm of Hg, R.R - 22/min, Temp 101.3°F Gen: ‘AAOx3 (Alert, Awake and oriented to time place and person), in mild to moderate pain, Heart: 5, S, heard. No thrills/murmurs /gallops/rubs. Lungs: CTA B/L (Clear to auscultation bilateral) Abdomen: — Flat, no scars and pigmentations, BS are + in all 4 quadrants. Tenderness is present in" periumbilical, RLQ and LQ regions, Not distended. No rebound/guarding/organomegaly. CVA tenderness is negative. Psoas and obturator signs are ~ D/D: Investigations: Pelvic Inflammatory disease Rectal and pelvic examination Pelvic abscess, CBC with differential Urinary tract Infection Urinalysis including C/S Appencicitis| Pregnancy test Rupture or Torsion of ovarian cyst Ultrasound Abdomen 8 Page 1 of 1 27 Yr. O/F complaining of rash Vitals « Pulse--78/min # B.P--120/75 mm of Hg + Temp--98.3 F = Rerate-22/min Make a mental checklist of Differential Diagnosis « Infections « Insect borne diseases + SLE « Photo dermatitis, # Drug induced + Occupational exposure » Rheumatoid arthritis, « Other autoimmune diseases ow Page tof t SP‘s notes If doctor asks you anyt lite. g other than these just say ‘no’ (or) say things that are normal # You are a 27y/o female c/o rash since 7 days on face and neck. © Ita flat rash appeared after gardening for 3 hrs. « Remained same as a flat rash but is increasing in size day by day * Increases on exposure to sun + No relieving factors * No new areas were involved «No itching/burning # No redness of eyes * No tenderness/no numbness + You also have joint pains since 4 days, early morning stiffness + Have fever Since 2 days + No breathing prablems/no chest pain « None of the femity members or close contacts has similar problems * No h/o travel + No history of similar past episodes ‘« Allergic to penicillin ‘+ Past h/o joint stiffness several times, subsides on its own + Never hospitalized «No urinary and G.1 problems « Family—mother has rheumatism « Obg/gyn—Has never been pregnant, last menstrual period was 2 weeks ago. «Sexually active with boyfriend, using condoms regularly ‘+ No smoking, ne alcohol «No recreational drugs. Took aspirin for headache 7 days ago ‘* Occupation—works in chemical manufacturing company S.P's Check List History Taking ‘Asked about : location of rash- whether the rash was initially fat or raised/blistered whether the rash change to any character- any new area involved- the progression aggravating and relieving factors: fever itching, burning, pain or numbness over the rash eyes redness, joint pain, recent travel. animal contact, Past History Asked about : similar episodes in the past: histoi high BP, diabetes, joint problems) previous hospitalization ¢ surgery), tamily health urinay and bowel problems appetite and change in weight. smoking, alcohol, ob/gyn history sexual history, medications, occupation and stress in life, ificit drug Examination Washed hands- looked inside mouth for oral ulcers- examined 3 joints- listened heart- uscuilated all over the lungs- examined face and neck for rash+ examined without gown. Counseling Explained physical findings and probable diagnosis- explained further work up ( blood test)- explained the importance of lifestyle modifications by quitting smoking, behavioral relaxation D.D for this ease SLE Rheumatoid arthritis Photo dermatitis, Drug induced Occupational exposure Infection Investigation CBC with differential count- ESR- antinuclear antibodies assay’ anti ds DNA- rheumatic factor assay- biopsy the rash skin tests for allergen of allergies: past medical problems | s Page 1 of 1 65 Yr. O/F complaining of arm and leg weakness Vitals Pulse--78/min » B.P--140/90 mm of Hg «© Temp-98.9F R.Rate--22/min Make a mental checklist of Differential Diagnosis > Stroke + Evolving stroke * Transient Ischemic attack = Hypoglycemia + Subarachnoid Hemorrhage «Intracranial mass ‘= Conversion disorder © Gullian Barre syndrome s Page 1 of 2 S.P's Notes If the doctor asks you anything other than these just say ‘no’ (or) say things that are normal in daily routine life. + You are a 65 year-old woman + You have noticed arm and leg weakness on the right side © it started an hour ago ‘+ The entire arm and leg fee! numb «= You are able to speak + You have noticed a gradual increase in the symptoms over the past one-hour «Nothing seems to make your symptoms better or worse No loss of consciousness + No bowel problems «No urinary problems «No fits/jerky movements « No headache and fever # No disturbance in vision # No history of verniting + You have been diagnosed with high blood pressure 25 years ago and you have not been taking medications regularly + You have had a heart attack (MI) 6 years ago after which you have undergone a bypass surgery + You smoke 2 packs of cigarettes per day and have been doing so since the past 40 years + You don't drink any alcoholic beverages except wine occasionally. 8 Page 2 of 2 « You are a widow. Your husband died 8 years ago. You don’t have any children * You live alone. « You have the neighbor Steve who is like a son to you. He brought you to the hospital. « You don't have a history of frequent falls. # You have never taken any recreational drugs. + You are not sexually active * You had a mother and father who both had high BP and both died because of a heart attack Ask this qt is he doesn’t address about stroke "Doc is it a stroke?” S.P's Cheek List History Taking ‘Asked about : the onset of weakness- the progression aggravating and relieving factors; asked how you most of the days belore weakness sensory changes- speech problem- dizziness: fever- loss of consciousness any jerky movement seizure: neck stiffness: vision changes: vomiting incontinence: history of falls: recent trauma Past History Asked about : similar episodes in the past: history of allergies: past medical probl high BP. diabetes, joint problems) previous hospitalization ( surgery), family health urinay and bowel problems appetite and change in weight, smoking. aleohol. sexual history. medications, occupation and stress in life, illicit drag Examination Washed hands- check pulses at two places- check cranial nerves 2-12 B/L.- function B/L- sensory- coordination and gait- check balance- did Romberg for cerebellar functions- did a heart and lung exam Counseling Explained physical findings and probable diagnosis- explained further work up- discussed smoking cessation and drug compliance- inquited if [need any help D.D for this ease Stroke Evolving stroke Transient ischemic attack Subarachnoid haemorrhage Conversion disorder Investigation CBC with differential count- periodic monitoring af vitals! neuro check- basic metabolic panel- CT sean of the head without contrast- doppler of the vataticls ns ( ed motor lest: test 8 Page 1 et! 29 Yr. O/F known sickle cell anemia pt c/o chest pain Vitals Pulse--98/min © B.P-120/75 mm of Hg Temp-101.3 R.Rate--22/min Make a mental checklist of Differential Diagnosis + Chest syndrome due to sickle cell anemia Pneumonia Costochondritis Salmonella Osteomyelitis, Panic attacks 8 Page 1 of | S.P’s Notes If the doctor asks you anything other than these just say ‘no’ (or) say things that are normal in daily routine life. » You are Mrs. Mary, age: 29yrs » Have chest pain since 12 hrs, » Started slowly, progressively increasing, + 7-8/10 in severity, « In midline near heart. * It's 9 type of sharp pain. «Moving around makes it worse, respiration worsens, OTC pain killers reduce the pain «Not associated with vomiting no nauseating feeling » No urine problems + No bowel problems « Last menstrual period was 2 weeks ago «Have fever since 3 days associated with chills and rigors, have cough associated with sputum which is green in color « H/O pain in fingers in past, h/o pain in abdomen in past + No allergies. ‘+ Once hospitalized far pain in abdomen and diagnosed as sickle cell anemia «Hes one sexual partner ‘¢ Using oral contraceptive pills + Families’ health mother suffered from Sickle cell disease, = Smoking ~ no + Alcohol- no + Occupation: Working as a teacher ‘+ Appetite is reduced and wt is normal «No illicit drug intake * No blood transfusion ‘¢ No exposure to hypoxic environment, dehydration or severe exercise «No trauma to chest S.P 's Cheek List History Taking Asked about : pain ( location, intensity, quality, the origin and duration, the progression. any radiation ) aggravating and relieving factors; any vomiting, fever, coug! ; SOB/dyspnea history of trauma; exposure to any hypoxic environmemt; h/o blood in urine; o of blood transfusion Past History Asked about : similar episodes in the past: history of allergies: past medical problems ( high BP. diabetes ) previous hospitalization ( surgery), family health, family Ivo sickle cel! appetite and change in weight, smoking, alcohol, ob/gyn history sexual history, medications, ovcupation Examination Washed hands- inspection of chest- palpation of area- Palpated abdomen superticially and deeply - examined without gown, Counseling lained physical findings and probable diugnosis- explained further work up ( blood test, chest x-ray) explained the complication of disease ( infeetion, hypoxia can precipitate the pain) D.D for this ease Chest syndrome due to sickle cell anemia Pneumonia Costochondritis Salmonella osteomyelitis Investigation CBC with differential count- Urinalysis- Stool culture for Saimonella- Chest ray- Spucuns e's hemoptys $ Page 1 of 1 25 Yr. O/M complaining of cough Vitals * Pulse--98/min © B.P--120/75 mm of Hg © Temp-101.3 © Rurate--22/min Make a mental checklist of Differential * Pneumonia + Pleurisy * Plural effusion © Tuberculosis Asthma Postnasal drip = Drugs # Allergic bronchitis = GERD S.P's Notes Page 1 of 1 If the doctor asks you anything other than these just say ‘no' (or) say things that are normal in daily routine life. You are Mr. Bill, age: 25yrs a paramedic, who drives an ambulance, have contact with patients. ‘You came with complains of cough since 3days,it is a productive cough, a teaspoonful of greenish sputum each time. No blood in the sputum Cough increases on lying down and decreases on sitting. Mild Problem with breathing, You also have chest pain which increases while taking deep breaths. Continuous fever since 2days with chills. No night sweats No history of weight loss No similar problems in the past. You are allergic to cats, Past history of sinusitis Never hospitalized No Urinary or bowel problems Families’ health is fine Sexually active only with your wife Smoking - 2 packs per day for 7yrs Alechol- 1 beer / day since 5 yrs Not taking any medications Occupation: Working as a paramedic S.P's Check List History Taking Asked about : onset of cough- duration: aggravating and relieving factors: whether it is dry or productive: any bload in sputum: describe sputum ( color, quantity); any problem w/ breathing: chest pain: fever: chills and rigor: comact w! ill persons Past History ‘Asked about : similar episodes in the past; history of allergies; past medical problems ( high BP. diabetes, TB. bronchial asthma ) previous hospitalization ( surgery). family health appetite and change in weight, smoking. alcohol, illicit drug intake: ‘occupation and travel history last PPD; urinary and bowel problems Examination Washed hands- inspected the breathing patiern- asked me to say 99 repeatedly ¢ palpated for tactical vocal fremitus)- tapped on my lung- Auscultated al over the lungs and heart- examined without gown, Counseling Explained physical findings and probable diagnosis- explained further work up ( blood lesi. chest x-tay. sputum studies) D.D for this ease Pneumonia Pleurisy Pleural cffision Tubercul Investigation CBC with differential count- sputum studies- chest x ray- PPD 8 Page 1 of 1 40 Yr. O/M complaining of fatigue and loss of weight Vitals © Pulse--78/min © B.P-120/75 mm of Hg + Temp-98.8F Rurate--22/min Make a mental checklist of Differential Diagnosis. * G.l malignancy © Hyperthyroidism + Depression * Diabetes mellitus * Anemia due to G.1 bleeding eHIV ‘Infections like TB + Addison's disease @ Page | of 2 S.P's Notes If the doctor asks you anything other than these just say ‘no’ (or) say things that are normal in daily routine life. «You are Mr. Albert, age: 40yrs ‘+ Have weakness and fatigue since 5 months * Started siowly, progressively increasing, fatigue Note: After the Doctor asked more about your complaint (or) any other complaints you should tell about the abdominal discomfort. «Noticed abdominal discomfort above the umbilicus + No jaundice, no vomiting, no fever «Stools are normal. # No interest in tife and other social activities « No history of forgetfulness « Thought that jife is not worth living + Appetite has reduced « Weight has reduced «30 pounds in last 3 months « Sometimes fee! palpitations (heart beats) « Temperature intolerance, feels hot most of the time. «No Edema of feet (No swelling) « Your sieep is decreased He gets up early in the morning + Lost his wife three months ago « Work place is very stressful ‘+ Constipation on and off © No allergies. * Have single sexual partner # Decreased libide «Has thought of suicide «Has not planned suicide + Has not attempted suicide + Decreased energy; feelings of guilt present © Decreased concentration ‘» Family health is normal Smoking - ne Alcohol takes couple of beer every evening Occupation: Working as a researcher No illicit drug intake Page ? of 2 S.P 's Check List History Taking Asked about : the weakness: any associated and fever- color of stools- thyroid problems- swelling in neck- bowel problem- change in appetite and weight- interest in lile- guilt feelings- plan. idea. attempt for suicide- difficulty in concentrating- inerease or decrease activity Past History Asked about : similar episodes in the past; history of allergies: past medical problems ( high BP. diabetes, kidney problems. urinary tract infection ) previous hospitalization ( surgery), family health appetite and change in weight, smoking, alcohol, obigyn history sexual history, medications, occupation Examination ‘Washed hands- examined eyes and oral cavity- neck- heart and lung auscultation- looked for tendon reflexes- mini mental status exam- 3-object command Counseling Explained physical findings and probable diagnosis- explained further work up ( blood ‘esi, thyroid profile. stool exam ) for this case G.1 malignaney Hyperthyroidism Depression Anemia due to G.l bleeding Investigation CBC with differential count- thyroid function tests- stool exam Jor occult blood- LF's- Uitrasound abdomen mptoms- asked about jaunclice- vomiting 30 Yr Page | of | 30 Yr. O/M came to HIV drug refill Vitals * Pulse-78/min B,P-120/75 mm of Hg + Temp-98.8F # Rerate-22/min 30 Yr Page 1 of 2 S.P's Notes If the doctor asks you anything other than these just say ‘no’ (or) say things that are normal in daily routine life. You are Mr. Nathan, age: 30yrs «You came for taking refill for Zidovudine. * You have been taking it for past six months « Taking 250 mg five times a day. Taking medications regularly. + No problems with taking medications (no muscle weakness etc...) «Not taking any other medications «It was diagnosed lyear back, ELISA testing further confirmed by Western blot. * No Cough # No fever, No history of night sweats, no headache, vomiting, no eye problems + No problems with swallowing ‘+ No swelling anywhere in the body No history of diarrhea «Vaccinations for Pneumonia taken last year + Appetite is reduced «Weight has reduced «No depression, you are fine ‘+ No white plaques in oral cavity +» No complaints of reddish papules over skin « Using only bottled water + No plans to travel in near future outside United States + No history of any ulcer/ discharge on genitalia. * No Allergies, » No history of tingling, numbness in extremity and pain in abdomen ‘* Have multiple sexual partners. All are males. «Using condoms. « Attitude towards life is positive. Have informed his sexual partners about his HIV status. «There is no one-take care of you, all your friends and family members abandoned you. ‘+ Families’ health is normal + Smoking - No «Alcohols No + Occupation: Working as a truck driver «No illicit drug intake + Not participating in any study S.P's Check List History Taking Asked about : the drug that are taking; about compliance: side effects of drugs: breathing, problems; fever: headache and vomiting; eyes problem- oral ulcers and white patches associated skin problem and rash: weakness and sensory problem in limbs: urinary and bowel problems Past History Asked about: history of allergies: past medical problems ( high BP. diabetes) previous hospitalization ( surgery). family health appetite and change in weight, smoking, alcohol, sexual history. medications. egcupation Examination Washed hands- palpated abdomen auseltated heart und lung- exam eyes w! ophthalmascope- check sensory system in hand and legs- examined without gown, Counseling, Explained physical findings- explained further work up ( blood test, viral titers. chest x ray)- explained the importance of safe sexual practives and use of condom D.D for this case HIV refill Investigation CBC with differential count- CD4 count- P24 antigen quantification- chest x ray LPTs. 30 Yr Page 1 of 1 35 Yr. O/F Came for evaluation of jaundice Vitals Pulse--98/min 8.P--120/75 mm of Hg Temp-101.3 R.rate--22/min Make a mental checklist of Differential Diagnosis for jaundice # Infectious hepatitis ‘+ Hemolytic jaundice # Alcoholic hepatitis, + Drug induced hepatitis + Primary biliary cirrhosis « Wilson's disease + Hemochromatosis + Malignancy 30 Yr Page 1 of 1 S.P's Notes If the doctor asks you anything other than these just say 'no' (or) say things that are normal in daily routine life. + You are Mrs, Catherine, age: 35yrs a housewife. = You came with complains of fever with chilis since Sdays. # You have noticed yellowish discoloration of eyes since 2 days. + You noticed pale colored stools and dark urine since 3 days. + There is 2 mild discomfort in the belly from a day. ‘+ Traveled to India and came back 20 days back. ‘+ Had blood transfusions 2 yrs ago for an accident (Trauma to legs), Has been hospitalized «No History of sore throat, Bleeding, or any enlarged glands. + No similar episodes in the past, + No specific allergies. « No history of high blood pressure, diabetes, hepatitis, liver disorders. + Appetite has decreased; there is no weight loss. * No problems with urinary and bowel habits. «Not a smoker. Consumes 3 beers / day since 20yrs + Took medication for fever. Did not take Hepatitis B vaccine, «Sexual history is fine and only with husband. Using condoms as contraception + No significant family history. Ask this qt "Doc is it a hepati S.P's Check List History Taking Asked about : the onset of jaundice: the duration; color of stool and urine: itching: abdominal pain; fever w/ chills: sore throat; any bleeding tendencies: enlarged gland: travel history Past History Asked about : similar episodes in the past: histary of allergi high BP. diabetes, hepatitis, liver disease) previous hospitalization ( surgery), family health urinary and bowe! problems: family health appetite and change in weight, smoking. alcohol. sexual history, medications, illicit drug Examination Washed hands- auscultated abdomen- palpated abdomen superticially and deeply- check rebound tenderness- palpated liver and spleen- examined lor enlarged nodes Counseling Explained physical findings and probable diagnosis- explained further work up ( blood test, LFTs, hepatitis screen) D.D for this case Infectious hepatitis Alooholie hepatitis Drug induced hepatitis Primary biliary cirrhosis past medical problems ( Investigation CBC with differential cownt- urine for bile salt anti-mitochondtial antibody- LFTs- hepatitis screen 80 Yr Page 1 of 1 26 Yr. O/M Complaining of Diarrhea Vitals * Pulse--98/min B.P--110/75 mm of Hg Temp-98.8F Rrate--22/min Make a mental checklist of Differential Diagnosis, « Gastroenteritis * Food poisoning + Malabsorption e HIV ‘* Inflammatory bowel disease * Irritable owe! disease * Drug induced 30 Yr Page 1 of 1 S.P's Notes If the doctor asks you anything other than these just say ‘no' (or) say things that are normal in daily routine life. » You are Mr. Smith, age: 26 yrs « Have diarrhea since 2 weeks, + Started after returning from trip to Guatemala © 6-7 times a day ‘* Loose motion haif to one cup each time «There is a sensation of incomplete evacuation and pain (tenesmus) + No blood in the stools ‘Foul smelling stools, float in the pan ‘+ Had similar episode 6 months back + No vornitings « Have mild abdominal discomfort # Feel very thirsty * You were diagnosed as HIV+ 2 yrs back. «No change in urination « No allergies. « Have multiple sexual partners out using condoms, infermed to all sexual partners about HIV status. + Most of them are males ‘# Used intravenous drugs in past Families’ heaith is normal © Smoking - No « Alcohol - takes couple of beers every week ‘+ Occupation: Working as a computer professional in San Francisco + Appetite is reduced and wt has reduced in last six months + Taking Drugs for HIV (Zidovudine 250 mg 5 times a day). S.P's Check List History Taking Asked about : the onset and frequency of diarrhea- amount of stool passed- consistency of stool associated pain and tenesmus- blood or mucous in stools- recent travel- vomiting fever- rashes Past History Asked about : similar episodes in the past; history of allergies, past medical problems ( high BP, diabetes, hepatitis, liver disease) previous hospitalization { surgery), family health urinary and bowel problems; family health appetite and change in weight, smoking, alcohol, sexual history, medications, ilicit drug Examination Washed hands- auscultated abdomen- palpated abdomen superficially and deepty- ‘examined skin for rashes- examined oral cavity- heart and lung exam Counseling. Explained physical findings and probable diagnosis- explained further work up ( blood test, stool exam)~ explained importance of safe sexual practices- advised me to take increased fluids D.D for this case Gastroenteritis Giardiasis Isospora infection Drug induced diarrhea Investigation CBC with differential count- stool c/s- stool exam for cyst and ova- CD4 count- P24 load 30 Yr Page 1 of 1 66 Yr. 0/M complaining of constipation Vitals + Pulse--78/min © B.P--120/75 mm of Hg * Temp-98.8F © Rurate--22/min Make mental checklist for constipation « Functional constipation + Bowel obstruction + Carcinoma of colon + Hypothyroidism + Hemorrhoids « Anal fissure « Autonomic neuropathy » Drugs «Stricture 30 Yr Page | ot 1 S.P's Notes If the doctor asks you anything other than these just say ‘no’ (or) say things that are normal in daily routine life. * You are Mr. Jansen, age: 66 yrs, + Have constipation on and off 8 weeks, « Have difficulty passing stools, they are painful = There is a sensation of incomplete evacuation, ‘+ You got similar complaints 3 months back but then it resolved on its own. + You use a lot of OTC constipation medicines. + You drink enough water but that has only increased frequency of urination + Sometimes there is bload in the stools and it is painful. + Do not have episedes of diarrhea in-between. + No complaints of vomiting and pain in abdomen * You have lost a lot of weight in past 2 months. Appetite is reduced too. + You feel fatigued + You feel sweaty when you take just one flight of stairs + Have a history of Hashimoto's thyroiditis and it resolved long ago + Has diabetes from the past 20 yrs and taking insulin * Your diet mainly has vegetable and very little of becon + No allergies. + No other medications ‘+ Not sexually active (because of old age} + Families’ health - fether died because of colon cancer ‘+ Smoking ~ 3 packets per day for 25 yrs + Alcohol- takes 2-3 glasses of wine every week * Occupation: Working as a supervising technician in Pharmaceuticals Company + Taking drugs for hypertension. + No feelings of depression and fatigue and loss of interest in life S.P’s Check List History Taking Asked about : the onset and frequency of stools- whether it is complete or partial- amount of stool passed- consistency of stool- associated pain and tenesmus- blood or mucous in stools- vomiting- fever- urinary problem- thyroid problem- depression, Past History ‘Asked about : similar episodes in the past; history of allergies; past medical problems ( high BP, diabetes. ) previous hospitalization ( surgery), family health urinary and bowel problems; family health appetite and change in weight, smoking, alcohol, sexual history, medications, illicit drug Examination Washed hands- auscultated abdomen- palpated abdomen superficially and deeply- Counseling Explained physical findings and probable diagnosis- explained further work up ( blood test, stool exam and colonoscopy) D.D for this case Functional constipation Carcinoma of colon Hypothyroidism Diabetic autonomic neuropathy Bowel obstruction Investigation CBC with differential count- rectal exam and stool exam for occult blood- ‘colonoscopy- thyroid screen- blood sugar 30 Yr Page t of | 40 Yr. O/M Complaining of Impotence Vitals Pulse--78/min B.P--120/75 mm of Hg Temp-98.8F = Rurate--22/min Make mental checklist for Impotence + Diabetic neuropathy + Chronic alcoholism « Leriche syndrome Anxiety and other psychiatric disorders «Head injury with endocrine dysfunction # Spinal cord dysfunction «Drugs like antihypertensives 30 Yr Page 1 of 1 S.P's notes If the doctor asks you anything other than these just say ‘no’ (or) say things that are normal in daily routine life. ‘+ You are not able to get erections since 3 months. It is gradual in onset. # Desire to do sex is present all the time. . » Aggravated by - stress « Relieved by - No relieving factors + No associated problems. + No previous episodes of sexual dysfunction ‘+ Sometimes you have problem with nocturnal erections ‘* No pain in the Jegs or thighs ‘+ You have your wife and one other sexual partner and you are having problems with both of them. «No previous treatment/evaluation = Have diabetes since 5 yrs, maintaining only with exercise and diet, not taking medications, last blood sugar test was 2 yrs ago. «Have high blood pressure ‘« Since 3 yrs taking atenolol, No other medications. « You also have generalized anxiety disorder since 5 yrs for which you are taking buspirone + You met with an accident 3 months back and you have been hospitalized for head trauma and spine trauma evaluation. But everything was fine. + No thyroid problems. «Smoking occasional * Alcohol 2-3 beers/day for 25 yrs. + Never took recreational drugs S.P's Check List History Taking Asked about the problem and when did it start- any problem of sexual desire- ejaculation problem- whether it is continuous or intermittent- whether it is sudden or progressive aggravating and alleviating factors- asked about nocturnal erection- any pain in the legs- anxiety and depression- thyroid problem Past History Asked about : past history of sexual problem; history of allergies; past medical problems (high BP, diabetes..) previous hospitalization ( surgery) or trauma, family health urinary and bowel problems; sleep problem: family health appetite and change in weight, smoking, alcohol, sexual history, medications, illicit drug Examination ‘Washed hands- palpated abdomen gently- examined pulsations in lower limbs ‘Counseling Advise home glucose monitoring. strict to diabetic diet- exercise program- quitting smoking and aleohol- D.D for this case Diabetic neuropathy Chronic alcoholism Anxiety Head injury w/ endocrine dysfunction Spinal cord dysfunction Investigation blood sugar- x ray spine- MRI of brain- rectal examination 30 Yr Page | of 1 Mother of 1 Yr. O/Baby With Fever Make a mental note of differential diagnosis of fever ¢ Respiratory tract infections Ear infections * Exanthematous diseases = Meningitis ‘© Urinary tract infections * Gastroenteritis, 30 Yr Page 1 of 1 S.P's Notes If the doctor asks you anything other than these just say ‘no’ (or) say things that are normal in daily routine life. + You are Mrs, Smith, age: 26 yrs * The 1-year-old baby is at home. Baby has fever since two days. + Itis high grade and the baby shakes once in a while but she is not drowsy or lethargic # Itnever touched normal ‘+ It responds to Tylenol + The baby has not been feeding since two days. + There is discharge from nose and ear. (Green in color) + The baby has been coughing since one day and breathing very fast since then * The baby has been passing yellow urine. And cries when he passes urine + Vomited twice which is purely the ingested food + No bowel problems * There has been one episode when baby has tremors + The whole body was jerking rhythmically and similarly on both sides * During the episode the baby lost urine ‘+ After the episode baby remained silent and irritable for a while then slept + You have been using cold water tepid sponging + The baby has completed ail the required immunization + The baby has been doing well in growing. * The baby can stand on his feet + Can say dada mama can hold things with his hand. * The baby was born at full term No complication in delivery. + The baby was breast-fed till 2 months and then formula was started, + There is no history of recent travel # No history of rashes + No history of exposure to any child with similar complaints # Elder siblings are doing well. No contacts with ill people S.P 's Check List History Taking ‘Asked about the onset of fever- progression- associated w/ chills- respond to medication- any episode of seizures- description of seizure- what happen after seizure, whether child \was irritable, was any part paralyzed- any cough, ear discharge- vomiting- urinary and bowel problems- exposure to infected person- rashes- immunization history- Past History ‘Asked about: similar episode in the past; history of allergies; past medical problems ( ear infection, convulsion, urinary tract infection..) previous hospitalization ( surgery) or trauma, family health Counseling Explained the differential diagnosis- explained the importance of child being here- explained further work up- advise to give fluids and antipyretic D.D for this case Febrile seizures Meningitis Acute otitis media w/ intracranial spread Acute urinary tract infection Investigation CBC with differential count- ESR- urinalysis- ear exam- fundoscopy- lumbar puneture 30 Yr Page 1 of 1 45 Yr. O/F Complaining of acute abdominal! pain in Rt upper quadrant Vitals Pulse--100/min « Temp--98.7 R,Rate--20/min 8.P--130/80 mm of Ka. Make a mental checklist for RUQ abdominal p * Acute Cholecystitis * Perforation of Peptic Ulcer © Pancreatitis © Appendicitis © Pneumonia # Acute Hepatitis © Rupture of Aortic Aneurysm 30 Yr Page 1 ot! S.P's Notes If the doctor asks you anything other than these just say ‘no’ (or) say things that are normal in daily routine life. + You are Mrs, Debbie, age: 45yrs ‘+ Have abdominal pain since 2 hrs + Started suddenly, progressively increasing, + 8-9/10 in severity, © Right upper quadrant » It’s.a stabbing type of pain. * Going to back of my scapula # Began after eating a large meal ‘+ Moving around, taking deep breath makes it worse «No alleviating factors « Associated with 2 episode of vomiting and sweating + Vomit contains yesterday food, ne blood ‘+ No Urinary or bowel problems No fever, No jaundice, Have mild cough since yesterday but no breathing problems. «Similar abdominal pain 5 months ago + No allergies. «Had peptic ulcer in the past and was treated 5 yrs ago «Hospitalized for detivery «Family health is normal © Obg/gyn- fine, LMP was 2 weeks ago ‘» Smoking - 1 pack per day for 20yrs ‘¢ Alcohol- 3 beers daily since 15 yrs + Sexual history--active with husband, no H/O sexually transmitted diseases in the past « Occupation- Working as @ house wife «Eats lot of fatty food '» Appetite and weight is nermal «Taking antacids for pain relief « No iNicit drug intake (cocaine) S.P's Check List History Taking ‘Asked about : pain ( location, intensity, quality, the origin and duration, the progression, any radiation ) aggravating and relieving factors; any vomiting, fever, urinary and bowel problems cough and breathing problem; jaundice Past History Asked about : similar episodes in the past; history of allergies; past medical problems ( high BP, diabetes, kidney problems, urinary tract infection ) previous hospitalization ( surgery), family health appetite and change in weight, smoking, alcohol, sexual history, medications, occupation, Examination Washed hands- Auscultated abdomen- Palpated abdomen superficially and deeply- ‘Check rebound tenderness- Looked for CVA tenderness- percussed for liver span- elicited Murphy's sign- auscultated lungs- Counseling Explained physical findings and probable diagnosis- explained further work up ( blood test, ultrasound, abdomen x-ray, pancreatic enzymes)- advise quitting smoking and aleoho! D.D for this case Acute cholecystit Perforation of peptic ulcer Pancreatitis Appendicitis Pneumonia Investigation CBC with differential count- Abdomen and chest x ray- Ultrasound abdomen- pancreatic enzymes- LFTs 90 Yr Page t of | 24 Yr. O/F Came for prenatal it for the first time Vitals * Pulse--78/min 4 B.P--120/75 mm of Hg + Temp-98.8F + Rerate--22/min 30 Yr Page 1 of 1 S.P's Notes If the doctor asks you anything other than these just say ‘no! (or) say things that are normal in daily routine life. Your last menstrual period was 12 weeks ago + Pregnancy was confiimed 6 weeks ago at home and hospital # This is your first pregnaney + You have not felt fetal movements yet * You don't have any H/C abortions + No H/O sexually transmitted diseases in the past + Your first menstrual period was at the age of 13 yrs, periods were regular, each cycle last for 4 to 5 days + No H/O + + Morning sickness/vemitings + Abdominal pain + Vaginal bleeding + Fever + Rash + Breathing problem + Swelling of feet «= You don’t have any other medical problems ‘+ No H/O blood transfusions in the past + You don't have any family history of birth problems in your family + No H/O urinary oF bowel related problems + You smoke cigarettes 1 pack per day since § yrs * You drink 1 beer per week since 3 yrs «Not taking any drugs + You are having adequate sleep + Not working since last month + Your husband is very co-operative with you + Taken rubeila immunization long back. S.P's Check List History Taking Asked about: LMP: asked how do I know about pregnancy; obstetrical history; ‘gynecologic history; any problem of pregnancy ( vomiting, fever, abdominal pain, vaginal bleeding ) Past History Asked about : history of allergies; past medical problems ( high BP, diabetes, kidney problems ) history of allergy and exposure to cats; history of sexually transmitted diseases, previous blood transfusion, urinary and bowel problem previous hospitalization ( surgery), family health appetite and change in weight, smoking, alcohol, working environment medications and illicit drugs, occupation Examination Washed hands- examined my eyes ( for pallor)- oral cavity ( for general hygiene). legs ( for edema)- auscultated heart and Jungs- examined and auscultated abdomen Counseling Explained physical findings and told me further work up { blood for Hb, TORCH screen, HIV ( took the consent), hepatitis screen, urinalysis, abdomen ultrasound }- advised me to stop smoking and alcohol- advised for safe sexual practices- Investigation CBC with differential count, blood grouping and typing- urinalysis Ultrasound abdomen- TORCH sereening- hepatitis B screening- HIV screening 30 Yr Page | of 1 60 Yr. O/M Complaining of shortness of breathe Is + Puls; -s0/min B.P--160/95 mm of Hg Temp-98.3 R.rate--26/min Make a mental checklist for shortness of breath 2 CCF ‘Ischemic Heart Disease * Cardiomyopathy * Bronchial asthma * Copp * Pleural effusion * Pneumonia © Preumetherax 30 Yr Page } of 1 S.P's Notes If the doctor asks you anything other than these just say ‘no’ (or) say things that are Normal in daily routine ti * You are Mr. Adam, age: GOyrs have shortness of breath (S.0.B) since 30 days + Initially only during strenuous work now gets aggravated even for mild to moderate exertion. *» You have nocturnat breathlessness, * 5.0.8 Is aggravated by ying down and mild to moderate exertion + Relieved by sitting and rest. * You also noticed swelling of your feet and ankles + No H/O chest pain or fatigue * Never heard of racing of your heart (palpitations) ‘+ No H/O fainting attacks but occasional dizziness is present. * Occasional dry cough + No similar problems in the past, + No allergies. + No H/o branchial asthma or any lung disease + Had high blood pressure for 20 yrs taking Atenolol 25 mg. * Never hospitalized + No Urinary or bowel problems * Families’ health - father died because of heart attack + Smoking - 1 pack per day for 30yrs + Alcohol - 1 glass of wine / day since 25 yrs * Occupation - Working asa fire fighter ‘+ Appetite and weight is normal * Diet - eats lot of junk and fatty food. # Tested for cholesterol 1yr ago and it was 300. SP 's Check List History Taking Asked about : the onset of $.0.B.- the progression aggravating and relieving factors, paroxysmal nocturnal dyspnea- orthopnea- cough- chest pain- palpitations- syncope and dizziness, Past History Asked about : similar episodes in the past; history of allergies; past medicat problems ( high BP, diabetes, bronchial asthma and other lung problems) previous hospitalization { surgery), family health urinary and bowel problems appetite and change in weight, smoking, alcohol, sexual history, medications, occupation and stress in life, illicit drug cholesterol level if | known Examination Washed hands- eye exam w/ ophthalmoscope- look for JVD- palpated for PMI- listened heart- auscultated all over the lungs- percussed liver span- check for pedal edema- check peripheral pulses- examined without gown, ‘Counseling Explained physical findings and probable diagnosis- explained further work up ¢ blood tests, chest x ray, EKG and echocardiogram) explained the importance of lifestyle modifications by quitting smoking, alcohol; D.D for this case CCF corp Ischemic heart disease Cardiomyopathy Pleural effusion Investigation CBC with differential count- chest x ray- EKG and Treadmill test- echocardiogram- pulmonary function tests 30 Yr Page 1 of 1 40 Yr. O/F with no energy and increased urination Vitals # Pulse--98/min + 8.P--100/65 mm of Hg +» Temp-98.3 R.rate--22/min Make a mental checkiist of Differential Diagnosis » D. Mellitus + Hyperthyroidism + Diabetes Insipidus * Psychogenic Polydypsia + Chronic Fatigue Syndrome + Cystitis (Urinary Tract Infection) ‘+ Diuretics and other drug Intake 30 Yr Page 1 of 1 S.P's Notes If the doctor asks you anything other than these just say 'no' (or) say things that are normal in daily routine life. «You are 4oyr old female feeling tired since 2 months * Increasing day by day «You also have increased urination-8-10 times /day «2-3 times in nights «Also have increased volume of urine «No burning urination, no urgency + Also have increased thirst--1 month ‘© Drinking lot of water and eating a lot these days. «You have also lost 12- 14 Ibs of wi/3 months ‘© No heat intolerance, tremors, sweating = You have a H/O minor trauma to your head -3 months back, just admitted and later discharged from the hospital with out any intervention. ‘* No similar problems in the past «No Allergies «No H/O High Blood Pressure and Diabetes. « Have Hashimoto's Thyroiditis and taking synthroid for 10 yrs. You don't have any thyroid related symptoms (Hair loss, Tremors, Voice change etc.) ‘* Not taking any other drugs including recreational drugs «No bowel problems ‘+ Family--father and mother have diabetes » No sexual problems No psychiatric problems ‘* No smoking, no alcohai consumption history «Occupation - plumber and no stress at work SP 's Check List History Taking Asked about: the onset of problem- frequency of urination- nocturia- the volume- burning urination, the urgency and hesitancy- the thirst- the water intake- the weight changes- trauma to head- psychiatric problem- thyroid symptoms Past History ‘Asked about : similar problems in the past; history of allergies, past medical problems ( high BP, diabetes, thyroid problems) previous hospitalization ( surgery), family health urinary and bowel problems appetite and change in weight, smoking, alcohol, sexual history, medications, occupation and stress in life, illicit drug Examination Washed hands- palpated thyroid- auscultated heart- tested muscle power, sensations, reflexes in both upper and lower limbs- tested for CVA tendemness- tested visual field and ‘examined fundus ‘Counseling Explained physical findings and probable diagnosis- explained further work up ( blood test, blood sugar, thyroid profile)- explained the importance of lifestyle modifications and. exercise program, D.D for this ease Diabetes mellitus Hyperthyroidism Diabetes insipidus Psychogenic polydypsia Chronic fatigue syndrome Investigation CBC with differential count- screening blood sugar- urinalysis- thyroid profile- serum electrolytes 30 Yr Page 1 of 1 25Yr. O/F Complaining of sore throat Vitals © Pulse--98/min B.P--120/75 mm of Hg ‘Temp-101.3 Rurate-+22/min Make a mental checklist of Differential Diagnosis Infectious mononucleosis Viral pharyngitis Streptococcal pharyngitis Infectious tonsillitis Quinsy (Peritonsillar abscess} * Posterior nasal drip

You might also like